Vous êtes sur la page 1sur 161

More Advance Praise for

Cerebrovascular Disease
“I am impressed with this unique format of case reports that are each instructive
of a clinical lesson, including modern brain imaging and intravascular approaches
to the diagnosis and treatment of brain lesions. The numerous figures are ample,
clear, and helpful. This little book will find its way in to the pockets of residents
everywhere.”
—Lewis P. Rowland, MD, Neurological Institute, Columbia University
Medical Center, New York, NY

“Dr. Chong’s book provides a thorough review of vascular neurology, all presented
through a series of quickly digested real-life cases. Each case illustrates a common
stroke-related diagnostic or treatment situation, and provides insight into the
approach of an experienced clinician. The rationale for decision-making is fully jus-
tified by the available evidence base where data exist, and by sophisticated reasoning
in the gray areas where there are no data. Novices will appreciate the fundamentals
of stroke neurology, but even experts will find new tidbits of information in this
easy-to-read but comprehensive book.”
—Mitchell S. V. Elkind, MD, MS, FAAN, FAHA, Fellowships Director,
Department of Neurology, Columbia University, New York, NY

“Stroke continues to be a major medical problem encountered by many health care


professionals, and it disables many of our patients. In the last few years, we have
made tremendous advances in the diagnosis, treatment, and prevention of stroke.
Dr. Ji Chong has put together this terrific book that instructs us, one case at a time,
about many of the evidence-based approaches to manage our stroke patients. As one
of my former stroke fellows at Columbia Presbyterian Medical Center, Ji provides
an energetic and fresh perspective in stroke care. Clinicians are constantly learning
from their patients and communicate with one another about patients every day. The
innovative approach in this book of weaving together the careful patient histories,
illustrative brain images, and other diagnostic images coupled with insightful com-
mentaries is a great way to learn about the latest approaches to stroke management.
Each case makes many teaching points and paints a picture to help us care for count-
less other patients in our daily pursuit to reduce the consequences of stroke.”
—Ralph L. Sacco, MD, Chairman of Neurology, Executive Director,
Evelyn McKnight Brain Institute, Olemberg Family Chair in Neurological Disorders,
Miller Professor of Neurology, Epidemiology and Public Health, Human Genetics,
and Neurosurgery, Chief of Neurology, Jackson Memorial Hospital, Miller School of
Medicine, University of Miami, Florida
What Do I Do Now?

S E R IES CO - E D I TO RS- I N - CHI EF

Lawrence C. Newman, MD
Director of the Headache Institute
Department of Neurology
St. Luke’s Hospital Center
New York, New York

Morris Levin, MD
Professor of Neurology and Psychiatry
Geisel School of Medicine at Dartmouth
Lebanon, NH

PR EVIO U S VO LU M ES I N THE S ERI ES

Headache and Facial Pain


Peripheral Nerve and Muscle Disease
Pediatric Neurology
Stroke
Epilepsy
Pain
Neuroinfections
Emergency Neurology
Cerebrovascular
Disease

Ji Y. Chong, MD
Director, Stroke Prevention Program
St. Luke’s Roosevelt Hospital
Assistant Clinical Professor of Neurology
Columbia University
New York

3
1
Oxford University Press is a department of the University of Oxford.
It furthers the University’s objective of excellence in research, scholarship,
and education by publishing worldwide.

Oxford New York


Auckland Cape Town Dar es Salaam Hong Kong Karachi
Kuala Lumpur Madrid Melbourne Mexico City Nairobi
New Delhi Shanghai Taipei Toronto

With offices in
Argentina Austria Brazil Chile Czech Republic France Greece
Guatemala Hungary Italy Japan Poland Portugal Singapore
South Korea Switzerland Thailand Turkey Ukraine Vietnam

Oxford is a registered trademark of Oxford University Press in the UK and certain other
countries.

Published in the United States of America by


Oxford University Press
198 Madison Avenue, New York, NY 10016

© Oxford University Press 2013


All rights reserved. No part of this publication may be reproduced, stored in a
retrieval system, or transmitted, in any form or by any means, without the prior
permission in writing of Oxford University Press, or as expressly permitted by law,
by license, or under terms agreed with the appropriate reproduction rights organization.
Inquiries concerning reproduction outside the scope of the above should be sent to the
Rights Department, Oxford University Press, at the address above.

You must not circulate this work in any other form


and you must impose this same condition on any acquirer.

Library of Congress Cataloging-in-Publication Data


Chong, Ji Y.
Cerebrovascular disease / Ji Y. Chong.
p. ; cm.—(What do I do now?)
Includes bibliographical references and index.
ISBN 978–0–19–990787–8 (pbk. : alk. paper)
I. Title. II. Series: What do I do now?
[DNLM: 1. Stroke—diagnosis—Case Reports. 2. Stroke—therapy—Case Reports. 3. Cerebrovascular
Disorders—diagnosis—Case Reports. 4. Cerebrovascular Disorders—therapy—Case Reports. WL 356]
616.8′1—dc23
2012042265

The science of medicine is a rapidly changing field. As new research and clinical experience broaden our
knowledge, changes in treatment and drug therapy occur. The author and publisher of this work have checked
with sources believed to be reliable in their efforts to provide information that is accurate and complete, and in
accordance with the standards accepted at the time of publication. However, in light of the possibility of human
error or changes in the practice of medicine, neither the author, nor the publisher, nor any other party who has
been involved in the preparation or publication of this work warrants that the information contained herein
is in every respect accurate or complete. Readers are encouraged to confirm the information contained herein
with other reliable sources, and are strongly advised to check the product information sheet provided by the
pharmaceutical company for each drug they plan to administer.

9 8 7 6 5 4 3 2 1
Printed in the United States of America
on acid-free paper
Dedicated to my patients, who continue to teach me about resilience
and determination. Bud Rowland, who taught me so much about
neurology, writing, and thinking, and to “never appeal to authority.”

Hwa and Hyun Chong; Bonnie and David Cantor; Kevin, Sam, and
Andrew Noah Cantor, for their constant support.
This page intentionally left blank
Preface

Nearly 800,000 people have a stroke each year in the United States. On
average, this means someone has a stroke every 40 seconds. Stroke is the
leading cause of disability in this country. These numbers add up to a huge
public health problem that needs to be tackled from all directions: preven-
tion of first stroke, diagnosis and acute treatment of stroke, and prevention
of recurrent strokes.
Neurologists, and specifically vascular neurologists, are the specialists
who primarily treat patients with cerebrovascular disease. But the sheer
number of patients with stroke, as well as the multiple risk factors for
stroke, guarantee that physicians of all disciplines will see a stroke patient.
Although a couple of unusual cases are presented, most of this book deals
with common, practical questions clinicians will encounter. Case presenta-
tions will highlight evidence-based practice. There are clinical trial data to
support many recommendations, but there are also areas in cerebrovascular
disease treatment that are still uncertain. Usual practice in the absence of
strong data will be noted as such.
Cerebrovascular disease is an exciting and evolving field. Many new ther-
apies and risk factors are being investigated, and there is much promise for
significant advances in the coming years.
This page intentionally left blank
Contents

1 Intravenous Tissue Plasminogen Activator (IV tPA) for Acute Ischemic


Stroke 1
IV tPA is the only FDA-approved treatment for acute ischemic stroke. Eligibility
criteria need to be reviewed carefully. It needs to be delivered in a timely fashion,
but select patients may be treated beyond the three-hour window, out to four and
a half hours.
2 Endovascular Treatment for Acute Ischemic Stroke 7
Select patients who are not eligible for IV tPA and are within an eight-hour
window may be treated with other acute endovascular therapies. Mechanical
thrombectomy with various approved devices is often considered in acute large
vessel occlusion, but the benefits have not yet been proven in clinical trials.
3 Combination Reperfusion Therapy for Acute Stroke 13
In patients who do not recanalize with IV tPA, adjunctive endovascular therapies
may be used to improve reperfusion in large vessel occlusion. Patients need to
be selected carefully. Registry data suggest some benefit, but further randomized
trials are needed.

4 Hemorrhagic Complications of tPA 21


Intracerebral hemorrhage is a known complication of tPA. Neurological change
after thrombolysis requires brain imaging to evaluate for hemorrhage. Different
hemostatic agents can be used for symptomatic hemorrhage. There is no
standardized regimen for this complication.
5 Stroke Mimic and Acute Treatment 25
Patients with stroke mimics who are eligible for tPA may safely be treated. The
diagnosis of a stroke mimic is typically made after the acute setting when the
window for acute treatment has closed. Thrombolytic treatment of a patient who
is subsequently diagnosed with a stroke mimic has not been associated with brain
hemorrhagic complications.

6 Minor Stroke Symptoms and Acute Treatment 27


Patients with minor stroke symptoms or rapid improvement of symptoms are at
high risk of worsening. Acute treatment with IV tPA is warranted if there is still a
residual deficit that is disabling. Waiting for resolution of symptoms removes the
possibility of some acute treatments.

7 Hemicraniectomy for Large MCA Stroke 31


Malignant MCA syndromes have high morbidity and mortality from edema, mass
effect, and herniation. Hemicraniectomy in select patients is life-saving, but patients
often have significant disability. Younger patients may have the most benefit.
8 Suboccipital Decompression for Cerebellar Stroke 35
Large cerebellar strokes can cause rapid neurological deterioration and death. These
patients need to be monitored closely for signs of hydrocephalus and brainstem
compromise. Surgical decompression is an effective, life-saving treatment.

9 Blood Pressure Management in Acute Stroke 39


Blood pressure is commonly elevated after a stroke. There are theoretical risks
of allowing very high blood pressures, but clinical data suggest early lowering of
blood pressure may worsen outcomes.
10 Primary Prevention of Stroke 43
There are many modifiable risk factors for stroke. Screening for risk factors in
the primary care setting and instituting early lifestyle modification or medical
treatment of risk factors will lower the risk of incident stroke.

11 Asymptomatic Internal Carotid Artery (ICA) Stenosis 49


Carotid stenosis is a risk factor for stroke. Revascularization of high-grade
asymptomatic carotid stenosis in select patients can lower the risk of incident
stroke. Endarterectomy and stenting are both associated with periprocedural
risk. Risks and benefits need to be evaluated carefully before proceeding with any
intervention.

12 Secondary Stroke Prevention After Lacunar Stroke 53


Lacunar strokes are strongly associated with hypertension. Long-term blood
pressure management is important after lacunar stroke. Antiplatelet therapy
should be instituted for secondary stroke prevention.

13 Secondary Stroke Prevention After Stroke Due to Carotid Stenosis 57


Patients with symptomatic carotid stenosis benefit from revascularization. The
risk of recurrent stroke is highest during the early period after a TIA or stroke.
Carotid endarterectomy and carotid stenting are options for treatment and
should be considered within the first two weeks if feasible.
14 Secondary Stroke Prevention After Stroke Due to Intracranial
Atherosclerosis 61
High-grade intracranial atherosclerosis is associated with a high risk of recurrent
stroke. Medical therapy with antiplatelet therapy and aggressive risk-factor
control is the preferred treatment regimen for stroke prevention in patients
with intracranial atherosclerosis. Stenting is associated with high morbidity and
mortality.

15 Secondary Stroke Prevention After Cardioembolic Stroke 65


Atrial fibrillation is a potent risk factor for stroke. Anticoagulation significantly
lowers recurrent-stroke risk in patients with atrial fibrillation. New oral
anticoagulants offer options in addition to warfarin.

x CONTENTS
16 Secondary Stroke Prevention After Cryptogenic Stroke with Patent
Foramen Ovale (PFO) 69
The presence of a PFO is associated with cryptogenic stroke, especially in young
patients. Medical therapy for patients with stroke and PFO include antiplatelet
therapy or anticoagulation. PFO closure is not routinely recommended for stroke
prevention.
17 Carotid Dissection 73
Carotid dissection should be considered as a stroke etiology, especially in
young patients with a history of trauma. Carotid dissection can be recognized
by exam findings and radiographic studies. Treatment can include aspirin or
anticoagulation.
18 Stroke in a Young Adult 79
Traditional vascular risk factors can contribute to stroke in young adults. In the
absence of typical risk factors in a young patient, a more extensive evaluation
is needed. Other more unusual causes of stroke can include autoimmune,
infectious, hematological, and toxic etiologies.

19 Mycotic Aneurysm Due to Bacterial Endocarditis 83


Bacterial endocarditis can cause neurological complications, including stroke,
hemorrhage, and mycotic aneurysms. Patients with endocarditis and neurological
complications need vascular imaging to evaluate for aneurysm.

20 Migrainous Stroke 89
Migraine may mimic stroke, but acute migraine can also be a (rare) cause of stroke.
Since this is a diagnosis of exclusion, other etiologies of stroke need to be investigated.

21 Dural Venous Sinus Thrombosis 93


Intracranial venous thrombosis is a rare cause of stroke. Diagnosis is made with
venous imaging such as an MRV. Investigation for underlying prothrombotic
state is needed. Treatment is typically with anticoagulation.
22 Moyamoya Disease 97
Moyamoya is a rare cause of stroke caused by distal carotid occlusion. Children and
adults may be affected. Surgical intervention may reduce the risk of recurrent stroke.

23 Reversible Cerebral Vasoconstriction Syndrome 101


Reversible cerebral vasoconstriction syndrome can cause thunderclap headache,
subarachnoid hemorrhage, and stroke. The clinical presentation can be similar to
aneurysmal SAH and therefore requires rapid angiography. Angiographic findings
of vasculopathy and resolution support this diagnosis.

24 Transient Ischemic Attack Diagnosis and Management 105


TIA is a risk factor for subsequent stroke. Some patients with TIA are at high risk
of early recurrence. Rapid evaluation and treatment reduce that risk.

CONTENTS xi
25 Hypertensive Intracerebral Hemorrhage 109
Hemorrhagic stroke is associated with high morbidity and mortality. A common
cause of ICH is hypertension. Hemorrhage due to hypertension usually occurs in
subcortical locations. Prevention involves aggressive blood pressure control.

26 Cerebral Amyloid Angiopathy 113


Cerebral amyloid angiopathy is more prevalent with advancing age. Recurrent
lobar hemorrhage, microhemorrhages, and white matter disease may be seen.
A presumptive diagnosis is made by clinical factors and imaging, and definitive
diagnosis is by pathology.
27 Intracerebral Hemorrhage Secondary to Arteriovenous
Malformation 117
AVMs may cause intracerebral hemorrhage. Treatment can be challenging,
and multimodal treatment is often required. The options include surgery,
endovascular embolization, and radiotherapy.
28 Intracerebral Hemorrhage (ICH) from Cavernous Malformation 121
Cavernous malformations may cause intracerebral hemorrhage and seizures.
Surgical resection is considered in patients with accessible lesions and recurrent
hemorrhage or refractory seizures.

29 Aneurysmal Subarachnoid Hemorrhage 125


Aneurysmal subarachnoid hemorrhage typically causes sudden severe headache.
Diagnosis is made by CT scan in most cases, but lumbar puncture may be needed
if CT findings are normal. Rapid evaluation and treatment are important because
of the high morbidity and mortality associated with rebleeding and vasospasm.

30 Perimesencephalic Subarachnoid Hemorrhage 131


Perimesencephalic SAH is a more benign subset of SAH. Clinical presentation
and CT imaging findings can be similar to aneurysmal SAH, so evaluation for
aneurysm is obligatory.
31 Asymptomatic Intracranial Aneurysm 135
Intracranial aneurysms may be detected incidentally on imaging. Location and
size of an intracranial aneurysm are helpful in stratifying risk of hemorrhage.
Risk stratification is important for treatment decision making. Treatment may be
observation, surgical clipping, or endovascular coiling.

Index 139

xii CONTENTS
1 Intravenous Tissue
Plasminogen Activator
(IV tPA) Treatment for Acute
Ischemic Stroke

Her INR and platelet count are normal. It is now 7:00 AM.
A 58-year-old woman with hypertension and diabetes
presents to the emergency room with right hemiparesis.
She had worked her usual night shift and gone home and
had fallen asleep at 3:30 AM. She awoke at 5:30 AM and
noticed right-sided weakness. She called EMS and arrived
in the ER at 6:20 AM. She has a blood pressure of 153/92.
She has normal speech and is without any evidence of
aphasia. She has a right facial droop. She has prominent
weakness of the right arm and leg. Her head CT scan
shows no evidence of hemorrhage or infarct (Figures 1.1,
1.2, 1.3 and 1.4).

What do you do now?

1
FIGURES 1.1–1.4 Non-contrast head CT showing basal ganglia calcifications but otherwise
normal findings.

T his woman is having an acute ischemic stroke. Her symptoms are


suggestive of ischemia to the left hemisphere, but there is no language
involvement, so a large-artery occlusion is less likely. A pure motor syndrome
involving the face, arm, and leg indicates a small, deep infarct. She was last
known normal at 3:30 am, and we are faced with a decision regarding acute
treatment at 7:00, which is 3.5 hours from onset (onset time is defined as
“witnessed onset” or “last known well”).
The benefits of IV tPA have been well established in patients with
ischemic stroke treated within 3 hours of symptom onset. The National
Institute of Neurological Disorders and Stroke (NINDS) trial of IV tPA

2 WHAT DO I DO NOW? CEREBROVASCULAR DISEASE


administered within 3 hours of acute ischemic stroke showed that there is
a significant improvement in outcome compared with placebo. The inclu-
sion and exclusion criteria for this trial form the basis for the inclusion and
exclusion criteria for clinical use of tPA (Table 1.1). Intravenous tPA is the
only FDA-approved treatment for acute ischemic stroke.
Unfortunately, the rates of tPA use are very low, and one of the reasons
is delay in presentation to the ER. There has been increased interest in try-
ing to extend the time window for treatment. A pooled analysis of throm-
bolysis clinical trial data suggested benefit of IV tPA out to 4.5 hours.
This led to the European Cooperative Acute Stroke Study (ECASS) 3 trial.
In this study, patients in the 3- to 4.5-hour window were randomized to
IV tPA 0.9 mg/kg (standard dose) versus placebo. There were additional
exclusion criteria beyond those from the NINDS trial: age greater than
80, any anticoagulant use (even with International Normalization Ratio
[INR]<1.7), National Institutes of Health Stroke Scale (NIHSS)>25, and
history of both prior stroke and diabetes. In the NINDS trial, 35% of
patients who received placebo recovered to complete independence, com-
pared with 50% with IV tPA. In the ECASS 3 trial, 45% recovered with

TABLE 1.1 Inclusion and exclusion criteria for IV tPA


Inclusion
Age >18
Clinical diagnosis of ischemic stroke with measurable deficit
Treatment can be initiated within 3 hours

Exclusion
Minor or rapidly improving symptoms
Seizure at onset
Stroke or head trauma within 3 months
Major surgery within 14 days
History of ICH
Aggressive treatment to lower BP<185/110
Subarachnoid hemorrhage (SAH)
Gastrointestinal (GI) or genitourinary (GU) bleeding bleeding within 21 days
Arterial puncture at noncompressible site
Heparin within 48 hours and increased Partial Thromboplastin Time (PTT)
INR >1.7
Platelets <100,000
Glucose <50 or >400

1 IV TPA FOR ACUTE ISCHEMIC STROKE 3


FIGURES 1.5–1.8 MRI diffusion-weighted imaging of the brain showing small scattered
infarcts in the left hemisphere.

placebo and 52% with tPA in the extended window, thus showing a ben-
efit even out to 4.5 hours. The biggest concern with IV tPA is brain hemor-
rhage risk. In ECASS 3, the symptomatic intracerebral hemorrhage (ICH)
rate was higher in the tPA group than in placebo, but it was comparable
to rates in patients treated within 3 hours. Overall, tPA appears beneficial
and without significant additional risk of ICH out to 4.5 hours. Although
it is not FDA-approved in the extended time window, the American Heart
Association and other societies have recommended tPA use to 4.5 hours
in select patients.
Post-tPA, blood pressure must be monitored and treated carefully,
with a goal BP<180/105. Patients cannot receive any antiplatelet or

4 WHAT DO I DO NOW? CEREBROVASCULAR DISEASE


anticoagulant medications for 24 hours, and invasive procedures must
be avoided.
In this patient, IV tPA was given at 7:20 am (3 hours and 50 minutes after
last known well), because she met ECASS 3 criteria for the extended win-
dow. The next day, she only had slight right-arm drift. Her follow-up imaging
showed no evidence of hemorrhage, and small infarcts on MRI (Figures 1.5,
1.6, 1.7 and 1.8). She was discharged home after two days in the hospital.

KEY POINTS

■ IV tPA is the only FDA-approved treatment for acute ischemic


stroke within 3 hours of symptom onset.
■ Patients benefit when treated early, but the window for treatment
may be extended to 4.5 hours in select patients.
■ Inclusion and exclusion criteria need to be reviewed carefully.
■ Blood pressure monitoring and control is important before and
after thrombolysis.

Bibliography
Adams HP Jr, del Zoppo G, Alberts MJ, et al. Guidelines for the early management of
adults with ischemic stroke: a guideline from the American Heart Association/
American Stroke Association Stroke Council. Stroke. 2007;38:1655–1711.
Hacke W, Donnan G, Fieschi C, et al. Association of outcome with early stroke treatment:
pooled analysis of ATLANTIS, ECASS, and NINDS rt-PA stroke trials. Lancet.
2004;363:786–774.
del Zoppo GJ, Saver JL, Jauch EC, Adams HP. Expansion of the time window for
treatment of acute ischemic stroke with intravenous tissue plasminogen activator:
a science advisory from the American Heart Association/American Stroke
Association. Stroke. 2009;40:2945–2948.
Hacke W, Kaste M, Bluhmki E, Brozman M, Dávalos A, Guidetti D, et al. Thrombolysis with
alteplase 3 to 4.5 hours after acute ischemic stroke. N Engl J Med. 2008;359:1317–
1329.
The National Institute of Neurological Disorders and Stroke rt-PA Stroke Study Group.
Tissue plasminogen activator for acute ischemic. N Engl J Med. 1995 ;333:1581–1587.

1 IV TPA FOR ACUTE ISCHEMIC STROKE 5


This page intentionally left blank
2 Endovascular Treatment for
Acute Ischemic Stroke

A 77-year-old man with hypertension, high cholesterol,


and coronary artery disease was hospitalized for
coronary artery bypass surgery. The surgery was
uneventful. Five days after surgery, he was seen by his
wife to suddenly stop talking. Onset time was clearly
noted to be 3:30 PM. He was found to have global aphasia
and right-sided weakness. A CT scan of the brain was
normal. Shortly after his scan, his symptoms improved,
with better speech, but he continued to make paraphasic
errors. His right-sided weakness improved to full
strength. At 5:00 PM, his symptoms worsened again, to
global aphasia and right-sided weakness.

What do you do now?

7
T his patient is exhibiting signs of a left middle cerebral artery (MCA)
occlusion. He has aphasia and right-sided weakness. He just had
coronary bypass surgery so is not eligible for IV tPA. The options for
treatment include intra-arterial (IA) thrombolysis, mechanical clot retrieval,
and medical therapy. There are data supporting all options.
The Prolyse in Acute Cerebral Thromboembolism Trial (PROACT) was
a large randomized trial evaluating the benefit of intra-arterial prouroki-
nase (proUK) in patients with MCA stroke treated within six hours. In
this trial, patients with MCA occlusion were taken for an angiogram and
randomized to intra-arterial infusion at the site of the clot with proUK
with heparin, versus heparin alone. Patients had inclusion and exclusion
criteria similar to those of the NINDS tPA trial, but there was an NIHSS
cutoff of 4 to 30. Forty percent of patients with IA proUK (vs. 25% of
controls) achieved a good outcome. Ten percent of patients in the treat-
ment arm (vs. 2% of controls) had symptomatic ICH. These results were
favorable for treatment, but a second randomized trial has not been done.
Therefore, this treatment is not FDA-approved. But based on the strength
of this trial and other, smaller studies, the American Heart Association
(AHA) recommends intra-arterial tPA within six hours as an option for
select patients with MCA occlusion, especially patients not eligible for
IV tPA.
Several device studies have been completed or are under way. The goal of
these trials was to determine effectiveness of mechanical thrombectomy in
removing clots in patients with ischemic stroke due to a large-artery occlu-
sion. Rather than infuse a lytic drug into a clot, various devices have been
studied to remove the clot. The Mechanical Embolus Removal in Cerebral
Ischemia (MERCI) study evaluated a corkscrew-like device that is deployed
into a clot. Once the clot is engaged, the device along with the clot is pulled
out to restore blood flow. The MERCI study, as well as subsequent studies
using this device, was not a randomized clinical trial. They evaluated rates of
recanalization compared with historic controls. They found recanalization
was associated with better outcomes.
Another study used the Penumbra device, a suction method to extract
clots. This was also a single-arm, non-randomized study. Solitaire With the
Intention For Thrombectomy Trial (SWIFT), the most recent trial, used a
stent-retriever device in which a stent is deployed into the site of occlusion

8 WHAT DO I DO NOW? CEREBROVASCULAR DISEASE


to immediately restore blood flow. The stent is then retracted to remove
the clot. This device was compared with the MERCI device in SWIFT.
This randomized trial found improved 90-day outcomes and mortality in
patients who were randomized to the stent device.
All of these interventions were performed in patients within an
eight-hour window. Although these devices are FDA-approved, they are
approved because these studies have shown that they are indeed effec-
tive in removing clots. Does this translate into good outcomes in stroke
patients? Recanalization is certainly associated with better outcomes. But
some patients may have good collateral blood flow and not need acute reca-
nalization. Or there may be irreversible infarction such that recanalization
will cause hemorrhage and worse outcomes. Patients who are older and
have multiple comorbidities may have more complications from treatment.
Some of these questions will be addressed by ongoing randomized clinical

FIGURE 2.1 Cerebral angiogram showing left upper division MCA occlusion.

2 ENDOVASCULAR TREATMENT FOR ACUTE ISCHEMIC STROKE 9


trials. In the absence of such data, the existing evidence suggests there is
a benefit to using mechanical thrombectomy in acute stroke, but patient
selection is extremely important.
In patients who do not receive thrombolysis or acute endovascular reca-
nalization, conservative medical therapy would include antiplatelet therapy,
statin institution, blood pressure optimization, and hydration. Clinical trials
have shown acute aspirin (ASA) was of benefit, along with heparin 5000 units
twice a day. Acute anticoagulation with heparin or low-molecular-weight
heparins is not recommended. There is risk of hemorrhagic transformation,
especially in a patient with a large infarct. Meta-analysis of anticoagulation
trials has shown no benefit in acute anticoagulation, mostly due to a signifi-
cant increased risk of hemorrhage.
This patient was taken emergently for a cerebral angiogram and was
found to have a left upper division MCA occlusion (Figure 2.1). He received
intra-arterial tPA with complete recanalization (Figure 2.2).
Post-intervention, he returned fully to normal and imaging showed no
infarct.

FIGURE 2.2 Complete recanalization of the occluded MCA branch after intra-arterial tPA.

10 WHAT DO I DO NOW? CEREBROVASCULAR DISEASE


KEY POINTS

■ There are several approved devices for removing clots in patients


with acute stroke.
■ There are no randomized trial data establishing the efficacy of
devices in improving outcomes, but select patients may benefit
from such therapy.
■ Intra-arterial tPA is an option for select patients with large-artery
occlusion if treated within six hours of stroke onset, and
mechanical thrombectomy may be an option in select patients
within eight hours.

Bibliography
Adams HP Jr, del Zoppo G, Alberts MJ, et al. Guidelines for the early management of
adults with ischemic stroke: a guideline from the American Heart Association/
American Stroke Association Stroke Council. Stroke. 2007;38:1655–1711.
Furlan A, Higashida R, Wechsler L, et al. Intra-arterial prourokinase for acute ischemic
stroke: the PROACT II study: a randomized controlled trial: Prolyse in Acute
Cerebral Thromboembolism. JAMA. 1999;282:2003–2011.
Penumbra Pivotal Stroke Trial Investigators. The Penumbra pivotal stroke trial: safety
and effectiveness of a new generation of mechanical devices for clot removal in
intracranial large vessel occlusive disease. Stroke. 2009;40:2761–2768.
Saver JL, Jahan R, Levy E, et al. Solitaire flow restoration device versus the MERCI
Retriever in patients with acute ischaemic stroke (SWIFT): a randomized
parallel-group, non-inferiority trial. Lancet. 2012;380:1241–1249.
Smith WS, Sung G, Saver J, et al. Mechanical thrombectomy for acute ischemic stroke:
final results of the Multi MERCI trial. Stroke. 2008;39:1205–1212.
Smith WS, Sung G, Starkman S, et al. Safety and efficacy of mechanical embolectomy in
acute ischemic stroke: results of the MERCI trial. Stroke. 2005;36:1432–1438.

2 ENDOVASCULAR TREATMENT FOR ACUTE ISCHEMIC STROKE 11


This page intentionally left blank
3 Combination Reperfusion
Therapy for Acute Stroke

A 69-year-old woman with hypertension and high


cholesterol presented to the ER two hours after onset
of acute right-sided weakness. Her initial blood pressure
was 220/105. She had normal language function. She
had a right facial droop and severe right-arm weakness.
Her right leg was moderately weak.
There is no evidence of hemorrhage or infarct, but
there is a dense left middle cerebral artery (LMCA)
sign indicating thrombus in the LMCA (Figures 3.1, 3.2,
3.3 and 3.4). She was given Labetolol 10 mg IV twice to
reach a blood pressure that would allow for treatment
with IV tPA. Her blood pressure came down to 175/95.
IV tPA was given at three hours post-onset.
After the tPA infusion, she developed gradual
worsening of her symptoms with increased word-finding
difficulty and dysfluency; her right arm became fully
paralyzed. Over the course of an hour, she became
globally aphasic. It was now five hours since the initial
onset of symptoms.

What do you do now?

13
T his patient also has a LMCA syndrome. Her head CT shows a dense
LMCA. Her initial event was suggestive of LMCA occlusion with
right-sided weakness. But she may have had reasonable collateral flow
maintaining perfusion to the cortex, initially sparing her language. However,
in order to administer IV tPA, we needed to lower the blood pressure.

FIGURES 3.1–3.4 Noncontrast head CT showing a hyperdensity in the left Sylvian fissure
consistent with a dense left MCA sign.

14 WHAT DO I DO NOW? CEREBROVASCULAR DISEASE


FIGURES 3.1–3.4 (Continued)

This may have precipitated the worsening symptoms by expanding the


ischemic penumbra, causing aphasia and worsening hemiparesis.
IV tPA only recanalizes large-artery occlusions in about 30% of cases.
Reocclusion is also common and has been seen in a third of patients after
tPA in transcranial Doppler (TCD) studies. In this patient, after a full
dose of IV tPA, she exhibited signs of worsening with a larger territory of

3 COMBINATION REPERFUSION THERAPY FOR ACUTE STROKE 15


ischemia. She was at five hours post-onset. Several studies have been done
evaluating recanalization of large-artery occlusions as described in the prior
vignette.
There have also been studies of “bridging therapy.” Its goal is to try
and recanalize the vessel as quickly and as effectively as possible. IV tPA
is readily available and quickly administered. This could then be followed
by intra-arterial therapy, which requires more specialized facilities and
operators. In large-vessel occlusions, some studies have evaluated the use
of 2/3-dose IV tPA followed by intra-arterial therapy. The rationale for
lower-dose systemic tPA is to lower the risk of hemorrhage, since addi-
tional local thrombolysis will be delivered endovascularly. Pilot studies
showed the feasibility and possible benefit from this approach. Registry
data of patients treated with the MERCI device suggests safety from a
hemorrhage standpoint in patients given full dose IV tPA, then treated
with mechanical thrombectomy. Clinical trial results for this strategy are
pending.

FIGURE 3.5 Cerebral angiogram A-P view showing left MCA stem occlusion.

16 WHAT DO I DO NOW? CEREBROVASCULAR DISEASE


A head CT done after tPA infusion showed no hemorrhage to account
for her worsening. We had already alerted the neuroendovascular sur-
gery team prior to the worsening, and the patient was in the angiog-
raphy suite at five hours post-onset. The angiogram revealed complete
occlusion of the LMCA. Intra-arterial tPA and mechanical thrombec-
tomy were performed, with full recanalization at 6.5 hours (Figures 3.5,
3.6, 3.7 and 3.8).
The next day, she had only a residual, mild right facial droop, and her
MRI showed scattered small infarcts in the LMCA territory (Figures 3.9,
3.10, 3.11 and 3.12).

FIGURE 3.6 Recanalization of occlusion after mechanical thrombectomy.

3 COMBINATION REPERFUSION THERAPY FOR ACUTE STROKE 17


FIGURES 3.7–3.8 Cerebral angiogram lateral view showing occlusion and recanalization.

18 WHAT DO I DO NOW? CEREBROVASCULAR DISEASE


FIGURES 3.9–3.12 MRI FLAIR sequence showing final infarct limited only to small portion of
the MCA territory.

KEY POINTS

■ Patients post-tPA require frequent neurological monitoring.


■ Worsening may be related to hemorrhage or extension of the
infarct, and needs to be evaluated by CT.
■ Randomized trials are needed to determine the benefit of the
bridging therapy approach for large-artery occlusion.

3 COMBINATION REPERFUSION THERAPY FOR ACUTE STROKE 19


Bibliography
IMS Study Investigators. Combined intravenous and intra-arterial recanalization for
acute ischemic stroke: The Interventional Management of Stroke Study. Stroke.
2004;35:904–911.
Lewandowski CA, Frankel M, Tomsick TA, et al. Combined intravenous and intra-arterial
rtPA versus intra-arterial therapy of acute ischemic stroke: Emergency
Management of Stroke (EMS) Bridging Trial. Stroke. 1999;30:2598–2605.
Smith WS, for the Multi MERCI Investigators. Safety of mechanical thrombectomy and
intravenous tissue plasminogen activator in acute ischemic stroke. Results of the
Multi Mechanical Embolus Removal in Cerebral Ischemia (MERCI) Trial, Part 1. Am J
Neuroradiol. 2006;27:1177–1182.

20 WHAT DO I DO NOW? CEREBROVASCULAR DISEASE


4 Hemorrhagic Complications
of tPA

A 92-year-old woman with hypertension and atrial


fibrillation who was independent at baseline developed
acute global aphasia, left gaze preference, and right
hemiplegia. She presented to the ER 45 minutes after
onset. A head CT showed no stroke or hemorrhage. Her
initial NIHSS was 22. She fluctuated over the course of
three hours with significant improvement, followed by
worsening. The fluctuations delayed treatment with IV
tPA, which was not administered until three hours post-
onset. She remained neurologically unchanged, and a
routine post-tPA CT was done the next day (Figures 4.1
and 4.2).

What do you do now?

21
T he most feared complication of IV tPA is intracerebral hemorrhage.
Asymptomatic hemorrhage can occur even without tPA. Hemor-
rhage within already infarcted tissue may not cause any symptoms.
Hemorrhagic transformation is more commonly seen in strokes due to
cardiac embolus.

FIGURE 4.1 Non-contrast head CT prior to IV tPA showing no hemorrhage but a basal ganglia
calcification.

FIGURE 4.2 Non-contrast head CT 24 hours post-tPA showing left MCA and ACA territory
infarct with hemorrhagic transformation.

22 WHAT DO I DO NOW? CEREBROVASCULAR DISEASE


It is important to distinguish asymptomatic hemorrhagic transforma-
tion from symptomatic hemorrhage. Symptomatic hemorrhage is when there
is blood visible on CT but also with some degree of clinical decline on
examination. Symptomatic hemorrhage has been defined differently in dif-
ferent trials. For example, in the NINDS trial, symptomatic hemorrhage
was defined as “hemorrhage seen on CT with suspicion of hemorrhage or
any decline in neurological status.” The ECASS 3 definition was “hemor-
rhage with more than 4-point change in the NIHSS or death attributable
to the hemorrhage.” It is important to note what measure of symptomatic
ICH is used, since it contributes to the different hemorrhage rates in trials.
In the NINDS trial, there was a tenfold increased risk of symptomatic ICH
after IV tPA (0.6% vs. 6%). ECASS 3 found 0.2% versus 2.4% rates, based
on their definition of symptomatic hemorrhage.
Predictors of symptomatic hemorrhage after tPA are: older age, higher
NIHSS, higher systolic blood pressure (SBP), and higher glucose levels.
Our patient had many of these predictors. However, higher likelihood
of hemorrhage should not limit treatment with tPA. This treatment has
been effective in different age groups and different levels of stroke sever-
ity. The mortality rates between tPA and placebo were equivalent in the
NINDS trial: patients may have hemorrhage and die, but they may also
die from an untreated stroke. The net benefit still favors treatment in
eligible patients.
There are no evidence-based guidelines for the treatment of hemorrhagic
transformation post-tPA. The number of patients who receive IV tPA is
relatively small, and the hemorrhagic complication rates are a fraction of
that. Of those, symptomatic ICH with hematoma expansion that can be
attributed to coagulopathy is exceedingly small, so a randomized trial of
hemostatic agents post-tPA would be very difficult. In the absence of clini-
cal trial data, we have empirically used cryoprecipitate, and if the prothrom-
bin time (PT) and partial thromboplastin time (PTT) are elevated, also
give fresh frozen plasma (FFP). The half-life of tPA is about five minutes, so
typically, when signs of hemorrhage become apparent, fibrinolytic activity
should be normal.
This patient was deemed to have asymptomatic hemorrhagic transfor-
mation since she had no neurological decline. She was monitored clinically,
and no hemostatic agents were used.

4 HEMORRHAGIC COMPLICATIONS OF TPA 23


KEY POINTS

■ Symptomatic ICH (SICH) after tPA was seen in 6% of patients in


the NINDS trial.
■ SICH typically occurs within the first 36 hours after treatment.
■ CT scan needs to be done acutely if there is neurological
deterioration, to evaluate for hemorrhage.
■ Correction of hematologic factors is not standardized.

Bibliography
Alshekhlee A, Mohammadi A, Mehta S, et al. Is thrombolysis safe in the elderly? Analysis
of a national database. Stroke. 2010;41:2259–2264.
Goldstein JN, Marrero M, Masrur S, et al. Management of thrombolysis-associated
symptomatic intracerebral hemorrhage. Arch Neurol. 2010;67:965–969.
Menon BK, Saver JL, Prabhakaran S, et al. Risk score for intracranial hemorrhage
in patients with acute ischemic stroke treated with intravenous tissue type
plasminogen activator. Stroke. 2012;43:2293–2299.

24 WHAT DO I DO NOW? CEREBROVASCULAR DISEASE


5 Stroke Mimic and Acute
Treatment

A 41-year-old woman with hypertension and migraines


presented with acute left arm weakness and numbness
and headache. She had migraines for five years with
holocephalic headaches but no aura. Three years
prior to presentation, she was hospitalized at another
hospital for left-sided sensory loss and dysarthria in
the setting of a migraine. She was told she had had a
stroke. She returned to normal after that event. The
day of this presentation, she noticed acute left arm
weakness and numbness without involvement of the
face or leg. On examination, her blood pressure was
initially 190/105 and spontaneously dropped to 151/98.
She was very uncomfortable from severe headache. She
had a flaccid, plegic left arm with anesthesia of the left
arm. She was within the window for IV tPA and had no
contraindications for thrombolysis.

What do you do now?

25
M igraine may be a stroke mimic. In the acute setting, there is little time
to complete a full evaluation and wait for resolution of symptoms if
tPA is being considered. Therefore, risks and benefits need to be carefully
reviewed and discussed with the patient. Hemorrhage rates from tPA
increase with higher NIHSS scores, since patients with larger strokes are
more likely to bleed with tPA. Conversely, patients with stroke mimics are
unlikely to have spontaneous ICH, since there is no brain infarct.
In one series of 512 patients treated with IV tPA, 14% were thought
to have a stroke mimic. The most common diagnoses for these patients
were seizure, migraine, and conversion disorder. None had hemorrhagic
complications with tPA.
This patient had a history of migraine, and she developed a focal motor
deficit in the setting of a migraine attack. On one hand, her young age and
history of migraine made complicated migraine a possibility. However, she
had reported a prior history of stroke. She also had hypertension. These
factors would increase the risk of recurrent stroke. Furthermore, she had
never had motor deficit with migraine before. Therefore, waiting to see if
symptoms improved to make a diagnosis of migraine would remove the
possibility of treatment, if this were indeed stroke.
The benefits outweighed the risks of treatment for this patient, and she
was given IV tPA. Her symptoms persisted for three days, then resolved. Her
MRI was subsequently normal and showed no evidence of recent stroke.
This patient was given the diagnosis of complicated migraine and treated
with migraine prophylaxis after a complete workup.

KEY POINTS

■ Patients with stroke mimics are likely to have a low risk for ICH.
■ The benefits outweigh the risks of treatment, since the diagnosis
of a stroke mimic typically occurs after the tPA window closes.

Bibliography
Chernyshev OY, Martin-Schild S, Albright KC, et al. Safety of tPA in stroke mimics and
neuroimaging-negative cerebral ischemia. Neurology. 2010;74:1340–1345.

26 WHAT DO I DO NOW? CEREBROVASCULAR DISEASE


6 Minor Stroke Symptoms and
Acute Treatment

A 57-year-old man with hypertension and high


cholesterol presented to the ER with acute right arm
weakness and right face numbness. He had no language
or speech involvement. While in the ER, his symptoms
improved, but he was still left with distal hand weakness
and clumsiness. He had no pronator drift and only slight
widening of the right palpebral fissure. His brain CT was
normal.

What do you do now?

27
O ften, patients with stroke who have rapidly improving symptoms do
not receive IV tPA. Similarly, patients with minor syndromes also do
not receive treatment. In the NINDS trial, patients were excluded from the
study with the following minor symptoms: pure sensory syndrome, isolated
dysarthria, isolated facial weakness, and isolated monoparesis with only
minor weakness.
Because these patients were excluded from the trial, one of the con-
traindications for tPA is mild or rapidly improving symptoms. However,
studies of patients who were not treated with IV tPA because of rapid
improvement or minor symptoms found these patients had worse out-
comes at discharge. In one study evaluating outcomes in 41 patients
who were deemed “too good to treat,” 27% died or were not able to
be discharged home because of worsening. They also found that patients
who had over 4-point improvement on the NIHSS were more likely
to have subsequent worsening. This is consistent with clinical observa-
tion that rapid improvement is often a more ominous sign than static
symptoms. Rapid improvement implies a dynamic state and is often fol-
lowed by worsening. If worsening occurs outside the IV tPA window, the
patient has lost his opportunity for treatment. Furthermore, as discussed
above, greater stroke severity is associated with greater hemorrhage rates.
Therefore, patients with minor syndromes should also be relatively safe to
treat from a hemorrhage standpoint.

FIGURES 6.1–6.4 MRI diffusion-weighted sequence showing small punctate infarcts in the left
frontal cortex.

28 WHAT DO I DO NOW? CEREBROVASCULAR DISEASE


FIGURES 6.1–6.4 (Continued)

This patient was young and worked as an engineer. Hand weakness would
have been disabling for him. For these reasons, this patient was treated with IV
tPA at 1 hour 55 minutes. His MRI showed two small punctate infarcts, with
one in the hand region of the motor cortex (Figures 6.1, 6.2, 6.3 and 6.4).

KEY POINTS

■ There is no NIHSS cutoff for IV tPA.


■ Minor symptoms in the NINDS trial included: isolated dysarthria,
isolated facial weakness, isolated sensory symptoms, and isolated
monoparesis with only minor weakness.
■ Rapid improvement may portend early worsening.

Bibliography
Rajajee V, Kidwell C, Starkman S, et al. Early MRI and outcomes of untreated patients
with mild or improving ischemic stroke. Neurology. 2006;67:980.
Smith EE, Abdullah AR, Petkovska I, et al. Poor outcomes in patients who do not receive
intravenous tissue plasminogen activator because of mild or improving ischemic
stroke. Stroke. 2005;36:2497–2499.

6 MINOR STROKE SYMPTOMS AND ACUTE TREATMENT 29


This page intentionally left blank
7 Hemicraniectomy for Large
MCA Stroke

A 69-year-old man with hypertension and atrial


fibrillation but who was not on anticoagulation was
evaluated for acute left hemiparesis. The patient was
found lying on the floor by the dog-walker. The time of
onset was unknown. He was brought to the ER and found
to have left neglect, right gaze deviation, left facial
droop, and left hemiplegia. His initial head CT showed
a dense right middle cerebral artery (RMCA) and early
infarct in the complete RMCA territory. He developed
increasing somnolence over the following two days, and
a CT scan three days after admission showed increasing
mass effect and midline shift (Figure 7.1).

What do you do now?

31
FIGURE 7.1 CT scan showing infarction of the complete RMCA territory with edema causing
compression of the right lateral ventricle and shift of midline structures to the left.

T his is a patient with a malignant RMCA stroke. He has clinical signs of


the full MCA territory being infarcted, and his imaging is consistent
with this. It is important to recognize malignant MCA syndromes, since
there is a very high association with early worsening and death (up to 80%).
Predictors of a malignant pattern include high initial NIHSS, usually over
16; younger age (less atrophy and room for swelling); and early signs of
edema.
The cascade of deterioration is due to edema. Edema is usually maximal
in 48 to 72 hours after a stroke. Because the skull is a rigid, confined space,
there is little room to accommodate extra mass from brain edema. Shifting
of normal brain occurs, as in this patient. His CT shows compression of
the right lateral ventricle and left shift with subfalcine herniation. The most
morbid herniation pattern is transtentorial. The uncus of the temporal lobe
can shift downward and compress the brainstem, leading to brainstem dys-
function and death.
Medical therapy includes intubation and ICU monitoring, prefer-
ably in a specialized ICU for neurological disease. Hyperosmolar therapy
with mannitol or hypertonic saline is often used to reduce brain edema.

32 WHAT DO I DO NOW? CEREBROVASCULAR DISEASE


The goal of these therapies is to reduce edema by drawing fluid intravas-
cularly. Hypothermia is another modality being evaluated for reducing
edema.
Another approach is hemicraniectomy. By removing the skull, edema-
tous brain contents may herniate outward rather than compress other vital
parts of the brain. There have been three European randomized trials of
hemicraniectomy in malignant MCA strokes. All three were stopped early.
A pooled analysis suggested benefit in patients treated within 48 hours of
stroke onset, under age 60. This analysis found that 78% of patients treated
medically died, versus 29% with hemicraniectomy. A recent Cochrane
meta-analysis also showed lower risk of death or disability, defined as
MRS>4, with hemicraniectomy. However, the outcome of death or MRS>3
was the same between the two arms. This suggests that patients may sur-
vive but with significant disability: unable to walk unassisted and requiring
assistance for most needs.
This patient was older. There was a discussion with the family, who
stated his prior wishes for life-saving treatments. He was taken for emer-
gency hemicraniectomy (Figure 7.2). He survived, but required continuous
care in a nursing home.

FIGURE 7.2 CT scan showing right hemicraniectomy with alleviation of mass effect on the
hemisphere.

7 HEMICRANIECTOMY FOR LARGE MCA STROKE 33


KEY POINTS

■ Hemicraniectomy can be life-saving in malignant MCA syndromes.


■ Patients under age 60 have been enrolled in trials, but an age
cutoff for surgery is not defined.
■ Quality of life after surgery requires further study.

Bibliography
Cruz-Flores S, Berge E, Whittle IR. Surgical decompression for cerebral oedema in acute
ischaemic stroke. Cochrane Database Syst Rev. 2012 Jan 18;1:CD003435.
Hacke W, Schwab S, Horn M, Spranger M, De Georgia M, von Kummer R. “Malignant”
middle cerebral artery territory infarction: clinical course and prognostic signs.
Arch Neurol. 1996;53:309–315.
Vahedi K, Hofmeijer J, Juettler E, Vicaut E, George B, Algra A, et al; DECIMAL, DESTINY,
and HAMLET investigators. Early decompressive surgery in malignant infarction of
the middle cerebral artery: a pooled analysis of three randomised controlled trials.
Lancet Neurol. 2007;6:215–222.

34 WHAT DO I DO NOW? CEREBROVASCULAR DISEASE


8 Suboccipital Decompression
for Cerebellar Stroke

A 47-year-old man with diabetes presented to the ER


with headache and vertigo. His initial brain CT was
normal. An MRI done 12 hours later showed a large
right cerebellar infarct with mass effect on the fourth
ventricle (Figures 8.1 and 8.2).
The next day, at 8:00 AM, he was noted to be alert,
but he had a right lower motor neuron facial droop,
normal strength, and right hemi-ataxia. At 9:40 AM, he
was found to be stuporous. He kept his eyes closed, was
inconsistently following commands, and had a left gaze
preference. His pupils were 1mm and sluggishly reactive.
He was intubated and mannitol 50 g given. A repeat
head CT showed the evolving right posterior inferior
cerebellar artery (PICA) infarct with effacement of the
fourth ventricle and acute hydrocephalus (Figures 8.3,
8.4, 8.5 and 8.6).

What do you do now?

35
FIGURES 8.1–8.2 MRI diffusion-weighted sequence showing large right cerebellar stroke.

FIGURES 8.3–8.6 CT scan showing mass effect on the fourth ventricle and new obstructive
hydrocephalus.

36 WHAT DO I DO NOW? CEREBROVASCULAR DISEASE


C erebellar strokes need to be recognized early. Similar to malignant MCA
syndromes, rapid deterioration can occur if the infarct is large. The
posterior fossa has even less space to accommodate edema. Worsening can
occur in several ways: edema with brainstem compression, compression of
the fourth ventricle and obstructive hydrocephalus, tonsilar herniation with
compression of the medulla, upward herniation of the vermis and midbrain
distortion, and compression of the SCA with extension of infarct.
Clinical deterioration has been shown to occur in three steps: (1) cer-
ebellar signs, (2) change in level of consciousness and signs of brainstem
compression, and (3) stupor/coma with posturing and autonomic dysfunc-
tion. This deterioration can be rapid.
Surgery is a treatment option. Surgery is typically a suboccipital craniec-
tomy to alleviate the pressure. In one case series of patients with cerebellar
strokes and significant mass effect, there was a significant difference between
patients treated with surgery compared with no surgery. Even 38% of
patients in coma and posturing were nondisabled at discharge with surgery.
Another nonrandomized study of 84 patients showed that half of patients
with worsening levels of consciousness treated with surgery had meaningful
recovery. However, there is a very short time window to intervene.
This patient had progressed to signs of brainstem compression and had
acute hydrocephalus from blockage of cerebrospinal fluid (CSF) at the level
of the fourth ventricle. He was immediately treated with hypertonic saline
and taken to the OR. He had a suboccipital craniectomy, and a ventricu-
lostomy drain was placed. Post-operatively, he was found to be alert and
following commands. He had a right facial droop only. Three months later,
he had the residual facial droop and was back at work.

KEY POINTS

■ Patients with large cerebellar strokes need to be monitored


closely.
■ If there are signs of deterioration related to increased mass
effect, surgical decompression needs to be considered, with the
goal of early treatment.
■ Even patients in coma and severe neurological deficit may benefit
from surgery.

8 SUBOCCIPITAL DECOMPRESSION FOR CEREBELLAR STROKE 37


Bibliography
Hornig et al. Space occupying cerebellar infarction: Clinical course and prognosis. Stroke.
1994;25:372–374.
Jauss M, Krieger D, Hornig C, et al. Surgical and medical management of patients with
massive cerebellar infarctions: results of the German-Austrian Cerebellar Infarction
Study. J Neurol. 1999;246:257–264.
Searls DE et al. Symptoms and signs of posterior circulation ischemia in the New England
Medical Center Posterior Circulation Registry. Arch Neurol. 2012;69:346–351.

38 WHAT DO I DO NOW? CEREBROVASCULAR DISEASE


9 Blood Pressure Management
in Acute Stroke

A 74-year-old man with hypertension, prior stroke, and


coronary artery disease who had had bypass surgery
20 years prior to presentation came to the ER with new
left-sided weakness. He was found to have new left face,
arm, and leg weakness. His blood pressure was 190/90.
He was out of the time window for IV tPA. His EKG
showed an old right bundle branch block but no ischemic
changes. The ER physician would like to treat his blood
pressure.

What do you do now?

39
B lood pressure and mortality after a stroke follow a J-shaped curve
with significantly low and high blood pressures associated with higher
mortality. Blood pressure is commonly elevated after a stroke, and there is
often an immediate impulse to treat the blood pressure. However, there
are several theoretical reasons to not treat. Cerebral autoregulation allows
for tight control of cerebral perfusion pressure over a wide range of mean
arterial pressures. However, the ischemic brain no longer has autoregulatory
capacity, and the cerebral perfusion pressure (CPP) is linearly dependent on
mean arterial pressure (MAP). Therefore, abrupt lowering of blood pressure
may expand the ischemic penumbra. Positron emission tomography (PET)
studies support this.
However, sustained high blood pressure is also associated with increased
mortality. Possible mechanisms may be increased edema or increased risk of
hemorrhage with elevated blood pressure.
In a small randomized trial, the investigators found early lowering
of blood pressure with labetolol or lisinopril, compared with placebo,
within 36 hours after stroke onset was associated with better outcomes
with treatment. Treated patients had SBP decrease of 21 mmHg, versus
11 in placebo. They found no early neurological deterioration, but there
was improved three-month mortality (9.7% vs. 20.3%). However, a much
larger randomized trial that included over 2,000 subjects did not find ben-
efit in acutely lowering blood pressure. This trial used candesartan ver-
sus placebo in ischemic or hemorrhagic stroke treated within 30 hours of
stroke onset. The mean difference in BP was 5/2 mmHg. There was no
benefit for early treatment, and in fact, there was a trend toward worse
functional outcome.
The AHA recommends treatment for SBP over 220 or diastolic blood
pressure (DBP) over 120. If treatment is warranted, the blood pressure should
be lowered cautiously, 15% to 20% in the first 24 hours. A short-acting,
easily titratable drug is preferred. Blood pressure often drops spontaneously
within the first 24 hours without any intervention. Certainly, if there is
evidence of end organ damage such as acute myocardial infarction (MI),
congestive heart failure (CHF), renal failure, hypertensive encephalopathy,
or retinopathy, early treatment may need to be instituted.

40 WHAT DO I DO NOW? CEREBROVASCULAR DISEASE


This patient’s blood pressure was not treated acutely, and 24 hours later
it had spontaneously decreased to 160/80. He remained neurologically
unchanged.

KEY POINTS

■ Blood pressure is commonly elevated in the setting of stroke.


■ Acute lowering of blood pressure is not routinely recommended
unless BP is >220/120.

Bibliography
Adams HP Jr, del Zoppo G, Alberts MJ, et al. Guidelines for the early management of
adults with ischemic stroke: a guideline from the American Heart Association/
American Stroke Association Stroke Council. Stroke. 2007;38:1655–1711.
International Stroke Trial Collaborative Group. The International Stroke Trial (IST): A
randomized trial of aspirin, subcutaneous heparin, both, or neither among 19435
patients with acute ischaemic stroke. Lancet. 1997;349:1569–1581.
Potter JF, Robinson TG, Ford GA, et al. Controlling hypertension and hypotension
immediately post-stroke (CHHIPS): a randomized, placebo-controlled, double-blind
trial. Lancet Neurol. 2009;8:48–56.
Sandset EC, Bath PM, Boysen G, et al.; SCAST Study Group. The angiotensin-receptor
blocker candesartan for treatment of acute stroke (SCAST): a randomised,
placebo-controlled, double-blind trial. Lancet. 2011 Feb 26;377(9767):741–750.

9 BP MANAGEMENT IN ACUTE STROKE 41


This page intentionally left blank
10 Primary Prevention of Stroke

A 65-year-old woman with high cholesterol, treated


for thyroid cancer four years ago, no history of
migraine, developed new-onset headache. She noticed
right frontal episodic sharp pain. She had an MRI
through her primary care doctor and was referred for
a stroke evaluation after MRI revealed an abnormality
(Figures 10.1 and 10.2). Her blood pressure was 130/80,
and she was neurologically normal.
She would like to know how to prevent a stroke.

What do you do now?

43
FIGURES 10.1–10.2 MRI FLAIR with white matter hyperintensities.

W ith the advent of more widespread imaging, we have been detecting


more white matter hyperintensities on MRI. One study in people
aged 60 to 90 found 18% had silent infarcts on MRI. Follow-up imaging
done (on average) three years later showed 14% had new infarcts, most of
which were also silent. Silent infarcts were associated with typical vascular
risk factors: increasing age, high blood pressure, diabetes, high cholesterol,
and smoking. White matter abnormalities are associated with an increased
risk of stroke and have also been associated with cognitive decline.

44 WHAT DO I DO NOW? CEREBROVASCULAR DISEASE


Although these white matter abnormalities were detected, this patient
did not have a clinical stroke. But because of the association between such
imaging findings and vascular risk factors as well as stroke, an evaluation
for vascular risk factors is warranted. She had imaging of her vessels and an
echocardiogram and cardiac monitoring. All results were normal.
In terms of primary prevention of stroke, patients should be screened for
risk factors (Table 10.1).
Hypertension is the most important modifiable risk factor for stroke.
The risk of stroke is directly related to the degree of hypertension. Multiple
trials have shown the benefit of blood pressure control in all age groups
and with all degrees of hypertension. There are numerous agents to choose
from, and angiotensin converting enzyme (ACE) inhibitors, diuretics, and
angiotensin receptor blockers (ARBs) have all been shown to lower the risk
of stroke. There may be race/ethnic differences in response to ACE inhibi-
tors, with one study showing higher stroke rates on lisinopril, possibly due
to less effective blood-pressure control in African-Americans. The choice of
agent can be tailored based on degree of hypertension, sociodemographic
factors, and the presence of other medical problems such as coronary dis-
ease or diabetes.

TABLE 10.1 Risk factors for stroke


Nonmodifiable risk factors for stroke:
Older age
Male sex
Race/ethnicity
Family history
Modifiable risk factors for stroke:
Smoking
Heavy alcohol consumption
Hypertension
Diabetes
High cholesterol
Atrial fibrillation
Coronary artery disease
Carotid stenosis
Hormone replacement therapy
Oral contraceptive use
Physical inactivity
Obesity

10 PRIMARY PREVENTION OF STROKE 45


All patients should have regular blood pressure monitoring and treat-
ment for blood pressure over 140/90, and over 130/80 if diabetic. Patients
should quit smoking, monitor serum lipids and glucose, and be counseled
on diet and exercise. Specifically for women, hormone replacement should
be avoided.
When risk factors are identified, patients often ask what their actual risk
of stroke is. To help with this, stroke risk-assessment scales have been devel-
oped to try to estimate an individual’s risk for stroke. These scales are based
on collecting baseline data from a specific study population and monitor-
ing them over time for the development of stroke. Because these scales are
based on the demographics of the population studied, the scale may not be
valid for a particular person. For example, a scale based on white men may
not be valid in predicting stroke for an African-American woman. Despite
such limitations, the Framingham Stroke Profile, for example, is a scale that
is widely used and includes a wide range of vascular risk factors. Points are
assigned for various risk factors. This scale, among others, is available online
for rapid calculation of risk.
The initiation of aspirin prophylaxis requires careful consideration of its
risks and benefits. Overall, ASA should be considered in patients who are at
high risk of having a cardiac event. ASA for primary prevention of stroke in
the general population is not well supported by epidemiological studies. In
women, 81 mg daily, or 100 mg every other day may lower the risk of isch-
emic stroke, especially in women over age 65. In otherwise healthy people
at low risk of a vascular event, ASA prophylaxis is not recommended.
For this patient, using the Framingham Stroke Profile, her age and blood
pressure would give her a 4% risk of stroke in 10 years. She was started on
ASA 81 mg daily, and her blood pressure was monitored closely.

KEY POINTS

■ Primary prevention involves careful monitoring and treatment of


vascular risk factors.
■ Aspirin for primary prevention of stroke is not recommended in
low-risk patients.

46 WHAT DO I DO NOW? CEREBROVASCULAR DISEASE


Bibliography
Chobanian AV, Bakris GL, Black HR, et al.; National Heart, Lung, and Blood Institute Joint
National Committee on Prevention, Detection, Evaluation, and Treatment of High
Blood Pressure; National High Blood Pressure Education Program Coordinating
Committee. The seventh report of the Joint National Committee on Prevention,
Detection, Evaluation, and Treatment of High Blood Pressure: the JNC 7 report.
JAMA. 2003 May 21;289(19):2560–2572.
Goldstein LB, Bushnell CD, Adams RJ, et al.; American Heart Association Stroke Council;
Council on Cardiovascular Nursing; Council on Epidemiology and Prevention;
Council for High Blood Pressure Research; Council on Peripheral Vascular Disease;
and Interdisciplinary Council on Quality of Care and Outcomes Research. Guidelines
for the primary prevention of stroke: a guideline for healthcare professionals
from the American Heart Association/American Stroke Association. Stroke. 2011
Feb;42(2):517–584.
Lawes CM, Bennett DA, Feigin VL, Rodgers A. Blood pressure and stroke: an overview of
published reviews. Stroke. 2004;35:776–785.
Rossouw JE, Anderson GL, Prentice RL, et al.; Writing Group for the Women’s Health
Initiative Investigators. Risks and benefits of estrogen plus progestin in healthy
postmenopausal women: principal results from the Women’s Health Initiative
randomized controlled trial. JAMA. 2002 Jul 17;288(3):321–333.
Seshasai SR, Wijesuriya S, Sivakumaran R, et al. Effect of aspirin on vascular and
nonvascular outcomes: meta-analysis of randomized controlled trials. Arch Intern
Med. 2012;172:209–216.
The ALLHAT Officers and Coordinators for the ALLHAT Collaborative Research
Group. Major outcomes in high-risk hypertensive patients randomized to
angiotensin-converting enzyme inhibitor or calcium channel blocker vs. diuretic:
the Antihypertensive and Lipid-Lowering Treatment to Prevent Heart Attack Trial
(ALLHAT). JAMA. 2002;288: 2981–2997.
Vermeer SE, Den Heijer T, Koudstaal PJ, et al. Incidence and risk factors of silent brain
infarcts in the population-based Rotterdam Scan Study. Stroke. 2003;34:392–396.

10 PRIMARY PREVENTION OF STROKE 47


This page intentionally left blank
11 Asymptomatic Internal
Carotid Artery (ICA) Stenosis

A 63-year-old man with hypertension, diabetes, high


cholesterol, tobacco abuse, and coronary artery disease
was referred for evaluation after being found to have
internal carotid artery stenosis. He denies every having
had a stroke or transient ischemic attack (TIA). He had
coronary artery bypass surgery five years ago and had
carotid Dopplers at that time. He thought they were
normal. He had a follow-up Doppler study recently
because of a carotid bruit detected on examination.
These Dopplers now show left internal carotid artery
stenosis greater than 80%. He would like to know if he
needs surgery.

What do you do now?

49
I n a patient with asymptomatic carotid stenosis, one of the questions is
whether revascularization is warranted. This is followed by what type of
revascularization should be considered. There are two large randomized
trials of carotid endarterectomy (CEA) versus medical therapy for stroke
prevention that support the use of CEA. In Asymptomatic Carotid
Atherosclerosis Study (ACAS), the angiographic criterion of >60% stenosis
was used. They found the five-year risk of ipsilateral stroke, perioperative
stroke, or death to be 5.1% with surgery, versus 11% with medical therapy.
The other trial, Asymptomatic Carotid Surgery Trial (ACST), included
Doppler criteria of 70% stenosis for enrollment. They found the five-year
risk of perioperative stroke, MI, death, or non-perioperative stroke to be
6.4% versus 11.8%. A follow-up study at 10 years showed the stroke risk to
be 10.8% in the CEA arm, versus 16.9% in the medical arm.
Overall, there appears to be a benefit of CEA in patients with asymp-
tomatic stenosis. But patients need to be selected very carefully. Subgroup
analysis suggests women may not benefit as much from CEA. Age also plays
a role, since the number of patients over age 75 in trials of surgery is too few
to establish its efficacy in older people. Life expectancy must be considered,
since there is an up-front risk of perioperative stroke and death before a
benefit may be realized. Choice of operator is also important. Any perioper-
ative complication rates greater than the 2.3% for stroke or death reported
in the ACAS trial could eliminate the potential benefit of the operation.
The other treatment option is angioplasty and stenting of the carotid
stenosis. The Carotid Revascularization Endarterectomy versus Stenting Trial
(CREST) was a randomized trial of symptomatic and asymptomatic patients
with carotid stenosis comparing stenting to CEA. In the asymptomatic patients,
stenosis was >60% by angiography or >70% by ultrasound. The primary out-
come was periprocedural stroke, death, or MI and ipsilateral stroke up to four
years of follow-up. They found no significant difference in outcomes (7.2%
CAS vs. 6.8% CEA). Perioperatively, stroke was more likely with CAS (4.1%
vs. 2.3%) and MI more likely after CEA (2.3% vs. 1.1%). Patients younger
than 70 did better with CAS, and those older than 70 did better with CEA.
These studies overall suggest a modest benefit to revascularization in pre-
venting stroke. However, medical therapy has improved over the years, with
better blood pressure control and wider use of statins. The benefit of CEA
or stent versus current aggressive medical therapy is not known. Patients

50 WHAT DO I DO NOW? CEREBROVASCULAR DISEASE


FIGURE 11.1 Conventional angiography showing high-grade left ICA stenosis.

should be selected very carefully for revascularization, and its risks and ben-
efits explained as discussed above.
This patient was young. He had been on good medical therapy after his
coronary bypass surgery and yet had significant progression of carotid dis-
ease. He also had coronary disease and multiple medical problems placing
him at higher risk for surgery. Given his age less than 70 and his comorbidi-
ties, he had a stent placed (Figures 11.1 and 11.2), without any periopera-
tive complications.

FIGURE 11.2 Cartoid stent placed, with improvement in stenosis.

11 ASYMPTOMATIC ICA STENOSIS 51


KEY POINTS

■ Select patients may benefit from revascularization of


asymptomatic high-grade carotid stenosis.
■ Carotid endarterectomy and endovascular stenting may be
considered, depending on the patient’s comorbidities and
operative risk profile.
■ Stroke rates may be lower now from carotid stenosis because of
aggressive medical management strategies.

Bibliography
Brott TG, Hobson RW, Howard G, et al. Stenting versus endarterectomy for treatment of
carotid artery stenosis. N Engl J Med. 2010;363:11–23.
Executive Committee for the Asymptomatic Carotid Atherosclerosis Study.
Endarterectomy for asymptomatic carotid artery stenosis. JAMA.
1995;273:1421–1428.
Halliday A, Harrison M, Hayter E, et al. 10-year stroke prevention after successful carotid
endarterectomy for asymptomatic stenosis (ACST-1): a multicentre randomized
trial. Lancet. 2010;376:1074–1084.
MRC Asymptomatic Carotid Surgery Trial (ACST) Collaborative Group. Prevention
of disabling and fatal strokes by successful carotid endarterectomy in patients
without recent neurological symptoms: randomized controlled trial. Lancet.
2004;363:1491–1502.

52 WHAT DO I DO NOW? CEREBROVASCULAR DISEASE


12 Secondary Stroke Prevention
After Lacunar Stroke

A 74-year-old man with hypertension arrives in the


office two months after a hospitalization for dysarthria
and ataxia. His hospital workup revealed a left pontine
lacune. He was started on ASA and simvastatin in the
hospital.
On examination, his blood pressure was 150/80.
He had a right facial droop, mild dysarthria, and slight
right arm drift. His MRI showed a left paramedian
pontine infarct. His MRA was normal. His transthoracic
echocardiogram was normal but had mild left ventricular
hypertrophy. His low density lipoprotein (LDL) was
40 and high density lipoprotein (HDL) 39 and his
liver function tests (LFTs) were normal. He is on an
angiotensin receptor blocker (ARB) for blood pressure.
He would like to ensure that he is on the best medical
regimen to prevent a recurrent stroke.

What do you do now?

53
L acunar strokes make up about 25% of all ischemic strokes. Lacunes are
caused by infarction of a small penetrator vessel. There are classic lacunar
syndromes: pure motor, pure sensory, sensorimotor, ataxic hemiparesis,
and clumsy-hand dysarthria. The sensitivity and specificity of these clinical
findings and a lacunar stroke are not one hundred percent, and a workup is
certainly needed to exclude large-vessel disease or a cardiac source of emboli.
The imaging finding of a small, deep infarct in the absence of other possible
etiologies is suggestive of a lacune. Lacunar strokes are strongly associated
with hypertension.
In this patient, a secondary stroke prevention strategy would need to
include aggressive blood pressure monitoring and treatment. Several studies
have shown that reduction in blood pressure significantly lowers recurrent
stroke risk. Although blood pressure targets need to be individualized based
on patients’ comorbidities, normotension is now defined as 120/80. The
choice of antihypertensives also needs to be individualized. The combina-
tion of diuretic and ACE inhibitor has been shown to be beneficial.
Statins are also important in secondary stroke prevention. The SPARCL
trial randomized patients with stroke or TIA but no coronary disease to
atorvastatin 80 mg versus placebo. Patients treated with atorvastatin had
lower rates of recurrent stroke and cardiac events. There did appear to be a
trend toward more hemorrhagic strokes.
Patients with lacunar strokes should also be on an antiplatelet agent.
Three medications may be used as first line of defense: aspirin, aspirin with
extended-release dipyridamole, and clopidogrel. There are minor differences
in efficacy, so choice of drug is also tailored to the patient. ASA is once a
day and is usually well tolerated. For chronic secondary stroke prevention,
doses of 50 mg to 1500 mg appear equivalent; however, there is greater gas-
trointestinal bleeding rates with higher doses. The typical dose prescribed
is 81 mg or 325 mg. The combination of aspirin with extended-release
dipyridamole when compared with ASA has a small benefit in secondary
stroke prevention. However, headache and gastrointestinal (GI) symp-
toms were more common. Clopidogrel and aspirin with extended-release
dipyridamole had similar efficacy in the Prevention Regimen for Effectively
avoiding Second Strokes (PROFESS) trial. Clopidogrel may have interac-
tions with proton pump inhibitors (PPI), which may increase the risk of
cardiovascular events. There are also genetic variants in hepatic metabolism

54 WHAT DO I DO NOW? CEREBROVASCULAR DISEASE


that may make clopidogrel less effective in some patients. But clopidogrel
may have less associated GI bleeding.
The Management of Atherothrombosis with Clopidogrel in High
Risk Patients with recent Transient ischemic attacks or ischemic stroke
(MATCH) trial showed that there were increased hemorrhagic complica-
tions in patients with stroke when placed on ASA and clopidogrel versus
clopidogrel alone, and there was no decrease in recurrent stroke rates. The
regimen of ASA and clopidogrel was specifically tested in patients with
lacunar strokes in the Secondary Prevension of Small Subcortical Strokes
(SPS3) trial. Patients with lacunes were randomized to ASA 325 daily ver-
sus ASA 325 and clopidogrel 75 mg daily. This trial showed no benefit of
dual therapy: recurrent stroke rate on ASA was 2.7% per year, versus 2.5%
per year on ASA and clopidogrel. There was a significant increased risk of
major hemorrhage and death on dual therapy.
Choice of agent may depend on cost, once- versus twice-daily dosing,
and tolerability of side effects. The combination of ASA and clopidogrel
should not be used routinely.
This patient was continued on ASA. His statin was changed to atorvas-
tatin. His blood pressure regimen was changed to increase his ARB, and
follow-up blood pressure was 120/60.

KEY POINTS

■ Preventing recurrent strokes after a lacune includes aggressive


blood pressure control.
■ Antiplatelet therapy should be instituted with either ASA, ASA
with extended-release dipyridamole, or clopidogrel.
■ Dual antiplatelet therapy with ASA and clopidogrel is associated
with increased risk of bleeding and death.

Bibliography
Amarenco P, Bogousslavsky J, Callahan A III., et al. High-dose atorvastatin after stroke or
transient ischemic attack. N Engl J Med. 2006;355:549–559.
Benavente OR, Hart RG, McClure LA, et al. Effects of clopidogrel added to aspirin in
patients with recent lacunar stroke. N Engl J Med. 2012;367:817–825.

12 SECONDARY STROKE PREVENTION AFTER LACUNAR STROKE 55


Diener HC, Bogousslavsky J, Brass LM, et al. Aspirin and clopidogrel compared with
clopidogrel alone after recent ischaemic stroke or transient ischaemic attack in high
risk patients (MATCH): randomized, double-blind, placebo-controlled trial. Lancet.
2004;364:331–337.
Diener HC, Cunha L, Forbes C, et al. European Stroke Prevention Study 2. Dipyridamole
and acetylsalicylic acid in the secondary prevention of stroke. J Neurol Sci.
1996;143:1–13.
Furie KL, Kasner SE, Adams RJ, et al. Guidelines for the prevention of stroke in patients
with stroke or transient ischemic attack. A guideline for healthcare professionals
from the American Heart Association/American Stroke Association. Stroke.
2011;42:227–276.
Progress Collaborative Group. Randomised trial of a perindopril-based
blood-pressure-lowering regimen among 6,105 individuals with previous stroke or
transient ischaemic attack. Lancet. 2001;358:1033–1041.
Sacco RL, Diener HC, Yusuf S, et al. Aspirin and extended-release dipyridamole versus
clopidogrel for recurrent stroke. N Engl J Med. 2008;359:1238–1251.
Wright JT, Dunn JK, Cutler JA, et al. Outcomes in hypertensive black and non-black
patients treated with chlorthalidone, amlodipine, and lisinopril. JAMA.
2005;293:1595–1608.

56 WHAT DO I DO NOW? CEREBROVASCULAR DISEASE


13 Secondary Stroke
Prevention After Stroke Due
to Carotid Stenosis

A 68-year-old woman with a history of tobacco abuse


presented with left-sided weakness and dysarthria. She
had been having some difficulty walking at home but
did not seek medical attention. Her family noted a facial
droop and brought her to the ER. She was noted to
have a left homonymous hemianopia, left facial droop,
mild left arm and leg weakness, and some sensory
neglect of the left. Her workup revealed an RMCA stroke
(Figure 13.1) and high-grade right internal carotid artery
(RICA) stenosis.

What do you do now?

57
FIGURE 13.1 MRI diffusion-weighted image showing right temporal and insular infarct.

P atients with stroke or TIA referable to a carotid artery with >70%


stenosis should be considered for revascularization. The North
American Symptomatic Carotid Endarterectomy Trial (NASCET) was an
important randomized trial of CEA versus medical therapy in patients with
symptomatic carotid stenosis. This study found a two year risk of ipsilateral
stroke of 26% in the medical arm versus 9% in the surgical arm (included
perioperative events). This is a dramatic absolute risk reduction, and CEA
had been the standard of care for years. Angioplasty and stenting is now also
an option, and there have been several randomized trials to support its use
in symptomatic carotid stenosis.
Most recently, CREST demonstrated no difference in the outcome of
stroke, MI, and death between patients (symptomatic and asymptom-
atic) treated with CEA versus stent. Looking specifically at the outcome
of stroke, stroke occurred more frequently with stent (4.1% vs. 2.3%). MI
occurred more frequently with CEA (2.3% vs. 1.1%). And there was an
age difference, with older people doing better with CEA and younger with
CAS (age 70 cutoff).
There are many factors to consider in the decision for surgery.
Operator experience is important. Surgical risk should be evaluated since
advanced-age, female gender, heart failure, active coronary artery disease,
lung disease, and chronic renal insufficiency have been associated with

58 WHAT DO I DO NOW? CEREBROVASCULAR DISEASE


perioperative complications with CEA. Patients with unfavorable anatomy,
neck irradiation, contralateral occlusion, and restenosis after prior CEA are
also at higher risk of perioperative stroke and death. With stenting, periop-
erative stroke may be more common than MI. Stenting also requires dual
antiplatelet therapy for the short term.
The highest risk period for recurrent stroke is within the first two weeks,
so when possible, patients should be treated within this window.
This patient had low surgical risk and with anatomy amenable to CEA.
She had surgery one week after her stroke.

KEY POINTS

■ Patients with symptomatic high-grade carotid stenosis with TIA or


non-disabling stroke benefit from revascularization.
■ CEA or stent may be considered, depending on the patient’s risk
profile and anatomy.
■ Treatment should be performed within two weeks if possible.

Bibliography
Barnett HJ, Taylor DW, Eliasziw M, et al. Benefit of carotid endarterectomy in patients
with symptomatic moderate or severe stenosis. North American Symptomatic
Carotid Endarterectomy Trial Collaborators. N Engl J Med. 1998;339:1415–1425.
Brott TG, Halperin JL, Abbara S, et al. ASA/ACCF/AHA/AANN/AANS/ACR/ASNR/CNS/
SAIP/SCAI/SIR/SNIS/SVM/SVS guideline on the management of patients with
extracranial carotid and vertebral artery disease: executive summary. Stroke.
2011;42:e420–e463.
Brott TG, Hobson RW, Howard G, et al. Stenting versus endarterectomy for treatment of
carotid artery stenosis. N Engl J Med. 2010;363:11–23.
Mas JL, Chatellier G, Beyssen B, et al. Endarterectomy versus stenting in patients with
symptomatic severe carotid stenosis. N Engl J Med. 2006;355:1660–1671.
Rothwell PM, Eliasziw M, Gutnikov SA, Warlow CP, Barnett HJ. Endarterectomy for
symptomatic carotid stenosis in relation to clinical subgroups and timing of surgery.
Lancet. 2004;363:915–924.

13 SECONDARY STROKE PREVENTION AFTER CAROTID STENOSIS STROKE 59


This page intentionally left blank
14 Secondary Stroke Prevention
After Stroke Due to
Intracranial Atherosclerosis

A 73-year-old man with hypertension and diabetes


was visiting his wife in the hospital when he developed
acute change in speech. He was noted to be speaking
gibberish and not making any sense. He had no vision,
motor, or sensory symptoms. The aphasia lasted one
hour and resolved spontaneously. In the ER, his blood
pressure was 180/113. He was neurologically normal. He
was admitted for an evaluation for TIA and was found
to have infarct on MRI despite resolution of symptoms
(Figure 14.1). His cardiac evaluation for source of emboli
was non-revealing. His CT angiogram (CTA) showed
intracranial atherosclerosis involving the LMCA with
irregularity and narrowing, but without focal stenosis
(Figure 14.2). He had evidence of atherosclerosis in the
basilar artery as well.

What do you do now?

61
FIGURE 14.1 MRI diffusion-weighted imaging showing patchy left MCA infarcts.

I ntracranial atherosclerosis is associated with a high risk of recurrent


stroke, especially with high-grade stenosis (70%–99%). Asians and
African-Americans are more likely to develop intracranial atherosclerosis.
The best preventative strategy in these patients has been debated in the past.
Warfarin-Aspirin Symptomatic Intracranial Disease Trial (WASID) was a
randomized trial of warfarin (INR 2–3) versus ASA 1300 mg in patients

FIGURE 14.2 CT angiography showing irregularity and stenosis of the left MCA.

62 WHAT DO I DO NOW? CEREBROVASCULAR DISEASE


with stroke or TIA with angiographically proven 50%–99% stenosis of an
intracranial vessel. Primary outcome was ischemic stroke, ICH, or vascular
death. This trial was stopped early because of a significant increase in
death and major bleeding in the warfarin arm without additional benefit,
compared with ASA. Medical therapy for intracranial atherosclerosis now
does not include anticoagulation.
Similar to trials of revascularization in extracranial carotids, there
have been studies of intracranial stenting to lower recurrent stroke rates
in patients with high-grade intracranial stenosis. Registry data suggested
there was a benefit. However, a randomized trial, Stenting and Aggressive
Medical Management for Preventing Recurrent stroke in Intracranial
Stenosis (SAMMPRIS) showed no benefit of intracranial stenting. In fact,
there were significantly higher rates of stroke and death at 30 days in the
stent arm (14.7% vs. 5.8%). One-third of the early strokes after stent were
hemorrhages.
Best medical therapy in this trial proved to be very effective. Medical
therapy included ASA 325, clopidogrel 75 mg for 90 days, aggressive blood
pressure control with a goal SBP <140 (or 130 if diabetic), LDL <70 with
a statin, diabetes treatment, smoking cessation, weight loss, and exercise.
The risk of stroke or death on medical therapy in this trial was less than
expected: 5.8% at 30 days and 12.2% at one year. It is not clear if the
short-term ASA and clopidogrel use had a role in the lower early stroke
rates. For now, aggressive medical therapy is the most effective option for
patients with intracranial atherosclerosis.

KEY POINTS

■ In most cases, intracranial atherosclerosis should be treated


medically.
■ Aggressive medical therapy is effective.
■ There is a high risk of stroke and death with intracranial stenting.

Bibliography
Chimowitz MI, Lynn MJ, Derdeyn CP, et al. Stenting versus aggressive medical therapy
for intracranial arterial stenosis. N Engl J Med. 2011;365:993–1003.
Chimowitz MI, Lynn MJ, Howlett-Smith H, et al. Comparison of warfarin and aspirin for
symptomatic intracranial arterial stenosis. N Engl J Med. 2005;352:1305–1316.

14 INTRACRANIAL ATHEROSCLEROSIS 63
This page intentionally left blank
15 Secondary Stroke Prevention
After Cardioembolic Stroke

A 55-year-old woman with hypertension and diabetes


was hospitalized with congestive heart failure and was
found to have rapid atrial fibrillation. Her heart rate was
controlled, and she was started on warfarin. She was
discharged home when her INR became therapeutic.
The next day, she developed acute facial droop and
dysarthria. She returned to the ER, and her INR on
admission was 2.4. She was found to have some mild
speech hesitancy and a right facial droop but was
otherwise neurologically normal.
Her MRI showed a left frontal infarct with some small
hemorrhagic transformation (Figures 15.1 and 15.2).
She was discharged on warfarin. She returned for
follow-up two weeks later and would like to discuss her
options for stroke prevention.

What do you do now?

65
A trial fibrillation is a potent risk factor for stroke. The CHADS2 score
is often used to risk stratify patients to determine treatment.

CHADS2:
C–CHF 1
H–Hypertension 1
A–Age > 75 1
D–Diabetes 1
S2–Stroke, TIA, or prior embolism 2

FIGURE 15.1 MRI FLAIR showing small left frontal infarct.

FIGURE 15.2 Gradient echo sequence with hypointensity consistent with hemorrhagic
transformation.

66 WHAT DO I DO NOW? CEREBROVASCULAR DISEASE


TABLE 15.1 Oral anticoagulants
Oral anticoagulants
Warfarin
Vitamin K–dependent
Multiple trials including European Atrial Fibrillation Trial (EAFT): recurrent
stroke 12% placebo vs. 4% with warfarin
Requires monitoring, dietary restrictions; may have poor compliance
Reversible, compliance can be monitored, inexpensive
Dabigitran
Oral direct thrombin inhibitor
Randomized Evaluation of Long term Anticoagulation Therapy (RELY):
Stroke or systemic embolism: 1.69% per year warfarin, 1.53% low-dose
dabigatran, 1.11% high dose;
Hemorrhagic stroke: 0.38% per year warfarin, 0.12% per year low dose,
0.10% per year high dose.
Higher rate of MI, no significant hepatotoxicity
FDA approved 150 bid, 110 mg dose not available in U.S., GI bleeding risk
especially in elderly; renal clearance, no reversal agent
Rivaroxaban
Factor Xa–inhibitor
Rivaroxaban Once Daily Oral Direct Factor Xa Inhibition Compared with
Vitamin K antagonism For Prevention of Stroke and Embolism Trial in
Atrial Fibrillation (ROCKET AF) stroke or systemic emboli: 2.2% per year
warfarin, 1.7% per year rivaroxaban; ICH: 0.7% warfarin, 0.5% per year
rivaroxaban
Higher CHADS2 score than in RELY
Once a day
Apixaban
Oral Factor Xa–inhibitor
Apixaban for Reduction in Stroke and Other Thromboembolic Events in
Atrial Fibrillation (ARISTOTLE): ischemic stroke: 0.97% per year apixaban,
1.05% per year warfarin
Hemorrhagic stroke 0.24% per year apixaban, 0.47% warfarin
Apixaban Versus ASA to Prevent Stroke in Atrial Fibrillation Patients
Who have Failed or are Unsuitable for Vitamin K Antagonist Treatment
(AVERROES): patients deemed unsuitable for warfarin, compared apixaban
with ASA; stroke or systemic emboli: 1.6% per year apixaban versus 3.7%
on ASA, ICH.4% in both arms
Not FDA-approved yet

15 SECONDARY STROKE PREVENTION AFTER CARDIOEMBOLIC STROKE 67


A score of 2 or higher typically warrants anticoagulation therapy. Since
patients who have had stroke automatically get 2 points, they are in a
moderate- to high-risk category and should be treated. Several studies in
the past have shown the benefit of chronic anticoagulation with warfarin
for secondary stroke prevention in patients with atrial fibrillation. For years,
warfarin was the only option for anticoagulation. Clinicians encountered
problems with compliance and difficulty with tight monitoring and control
of the INR with the narrow goal INR range of 2 to 3. Recently, new options
have been studied and approved by the FDA (Table 15.1).
All the new agents are as effective or more so in preventing stroke as
warfarin. They also appear safer from a hemorrhagic stroke standpoint. This
patient had a small stroke with a small hemorrhagic transformation. Two
weeks after the stroke, warfarin was stopped and she was put on dabigitran.

KEY POINTS

■ Anticoagulation is the most effective strategy to prevent strokes


in patients with atrial fibrillation.
■ Newer agents now provide more options for anticoagulation.
■ Dabigitran, rivaroxaban, and apixaban appear to be at least as
effective as warfarin and have fewer intracerebral hemorrhage
complications.

Bibliography
Connolly SJ, Ezekowitz MD, Yusuf S, et al. Dabigitran versus warfarin in patients with
atrial fibrillation. N Engl J Med. 2009 Sep 17;361(12):1139–1151.
Connolly SJ, Eikelboom J, Joyner C, et al. Apixaban in patients with atrial fibrillation. N
Engl J Med. 2011;364:806–817.
Granger CB, Alexander JH, McMurray JJ, et al. Apixaban versus warfarin in patients with
atrial fibrillation. N Engl J Med. 2011 Sep 15;365(11):981–992.
Patel MR, Mahaffey KW, Garg J, et al. Rivaroxaban versus warfarin in nonvalvular atrial
fibrillation. N Engl J Med. 2011 Sep 8;365(10):883–891.

68 WHAT DO I DO NOW? CEREBROVASCULAR DISEASE


16 Secondary Stroke
Prevention After
Cryptogenic Stroke with
Patent Foramen Ovale (PFO)

A 53-year-old woman with a history of hypothyroidism,


hypertension, diabetes, and high cholesterol
developed acute visual symptoms. She noticed a loss
of vision on the left side. She was initially seen by
an ophthalmologist, who found a left homonymous
hemianopia. She had an MRI, which confirmed a left
posterior cerebral artery (PCA)–territory stroke.
Her large arteries were normal. A transesophageal
echocardiogram revealed a patent foramen ovale.

What do you do now?

69
T his is a patient who had a stroke, and an extensive evaluation did not
reveal a large-artery or obvious cardiac source of emboli. The infarct
was not a lacune. An embolic-appearing stroke without an obvious source
would be classified as cryptogenic. About 30% of strokes are categorized
as cryptogenic, but in patients under age 55, 40% may be cryptogenic.
Several studies have suggested an association between PFO, or PFO and
atrial septal aneurysm, and cryptogenic stroke.
A patent foramen ovale is a persistent communication between the two
atria of the heart. This communication typically closes after birth, but in
about 20% of people, it remains open as a PFO. One possible mechanism
for stroke from PFO is a paradoxical embolus, in which a venous clot can
traverse the PFO and enter the arterial circulation to the brain. Although
studies have found an association between PFO and stroke, it does not
appear to confer additional risk of recurrent stroke compared with crypto-
genic stroke without PFO if the patient is on medical therapy.
The ideal medical therapy is not entirely clear. A subset of a larger ran-
domized trial of warfarin versus ASA in stroke evaluated patients with cryp-
togenic stroke and PFO. In the PFO in Cryptogenic Stroke Study (PICSS)
sub-study, there was no difference in recurrent stroke rates between patients
on ASA versus those on warfarin. However, this patient population was
older and had more vascular risk factors than other studies of patients with
cryptogenic stroke. The PFO may not have been a significant contributor
to stroke risk, so the results of this study may not apply to young patients
with PFO and no other vascular risk factors.
PFO closure has also been considered in patients with cryptogenic
stroke. The CLOSURE trial used a percutaneous occlusive device to close
PFOs. In this study, the two-year stroke rate was 2.9% in the closure arm,
compared with 3.1% in the medical arm; there was no statistically signifi-
cant difference. The medical arm received either ASA, warfarin, or both, at
the investigator’s discretion. The patients treated with PFO closure had a
higher incidence of post-procedure atrial fibrillation. Given the potential
complications and no clear benefit, PFO closure is not routinely recom-
mended for secondary stroke prevention.
Other studies using longer-duration cardiac monitoring suggest that there
is a higher rate of paroxysmal atrial fibrillation in patients with cryptogenic

70 WHAT DO I DO NOW? CEREBROVASCULAR DISEASE


stroke. It is important to remember to investigate other potential causes of
emboli thoroughly.
This patient was continued on medical therapy with ASA, statin, blood
pressure and glycemic control. She had ambulatory cardiac monitoring for
two weeks that did not reveal paroxysmal atrial fibrillation.

KEY POINTS

■ PFO is associated with cryptogenic stroke.


■ Medical therapy is favored for secondary stroke prevention, which
may include ASA or warfarin.
■ Patients with cryptogenic stroke should be investigated
thoroughly for other possible etiologies besides PFO.

Bibliography
Bogousslavsky J, Garazi S, Jeanrenaud X, et al. Stroke recurrence in patients with patent
foramen ovale: the Lausanne Study. Neurology. 1996;46:1301–1305.
Furlan AJ, Meisman M, Massaro J, et al. Closure or medical therapy for cryptogenic
stroke with patent foramen ovale. N Engl J Med. 2012;366:991–999.
Homma S, Sacco RL, Di Tullio MR, et al. Effect of medical treatment in stroke patients
with patent foramen ovale: Patent foramen ovale in Cryptogenic Stroke Study.
Circulation. 2002;105:2625–2631.
Mas JL, Zuber M. Recurrent cerebrovascular events in patients with patent foramen
ovale, atrial septal aneurysm, or both and cryptogenic stroke or transient ischemic
attack. Am Heart J. 1995;130:1083–1088.
Messe SR, Silverman IE, Kizer JR, et al. Practice parameter: Recurrent stroke
with patent foramen ovale and atrial septal aneurysm: Report of the Quality
Standards Subcommittee of the American Academy of Neurology. Neurology.
2004;62:1042–1050.

16 SECONDARY STROKE PREVENTION AFTER CRYPTOGENIC STROKE WITH PFO 71


This page intentionally left blank
17 Carotid Dissection

A 60-year-old healthy woman was brought to the


emergency room with acute onset of global aphasia,
left gaze deviation, and right hemiplegia. She had been
seen in the ER two days prior to presentation after being
struck by a car while walking on the street. She had a
concussion, scalp laceration, and hand fracture but was
discharged from the ER after normal CT of the head and
cervical spine. At home, she had episodes of positional
vertigo but was otherwise neurologically normal until
the morning of presentation. She had witnessed onset of
acute neurological change and was brought into the ER
within 45 minutes of symptoms. She was found to have
a left Horner’s syndrome and signs of a large left MCA
stroke.
Her head CT and CTA showed a dense LMCA sign
and absence of flow in the extra and intracranial left
ICA (Figure 17.1). She had no evidence of hemorrhage or
early infarct signs on the CT.

What do you do now?

73
FIGURE 17.1 CT scan showing dense left MCA sign, and CT angiogram showing occlusion of
the left ICA.

D issection of the cervical carotid artery must be considered in a patient


with history of trauma presenting with stroke and Horner’s sign. A
dissection is typically due to an intimal tear allowing blood extravasation
into a false lumen. This causes an intramural hematoma that can lead to
vessel stenosis as well as compression of other structures. Sympathetic
fibers run along the carotid artery so that a carotid dissection is associated
with sympathetic dysfunction (Horner’s syndrome): ptosis, miosis, and
anhydrosis. Anhydrosis is typically not present with carotid dissection
since the sudomotor fibers exit along the external carotid artery. Patients
with dissection also commonly have headache or neck pain. Hypoglossal
dysfunction can also occur.
Dissections can be traumatic, as in this case, or spontaneous. Traumatic
dissections may be due to minor trauma such as a fit of vomiting. Case
reports have also described dissection after chiropractic manipulation and
visits to the hair dresser with hyperextension of the neck. Spontaneous dissec-
tion has been associated with collagen vascular disease such as Ehler-Danlos
type IV, Marfan’s syndrome, and fibromuscular dysplasia. The etiology of
most spontaneous dissections is not found.
Diagnosis is usually by noninvasive imaging. MRI with cross-sectional
imaging of the vessel can detect the intramural hematoma. CTA can also
show crescentic enhancement.

74 WHAT DO I DO NOW? CEREBROVASCULAR DISEASE


Although IV tPA is not contraindicated in patients with dissection and
stroke, recent head trauma is a contraindication. Because acute endovascular
therapy could allow for early recanalization, she was treated with mechani-
cal embolectomy (Figure 17.2).
The prognosis is typically good with dissection. The majority of stenoses
from dissection resolve within the first two to three months. In one case
series of 161 patients with carotid dissection and severe stenosis or occlu-
sion, 50 had persistent stenosis at one year, versus 111 who recanalized.
The annual recurrent stroke rates in these patients were very low with 0.7%
in the patients with permanent stenosis versus 0.3% in the patients with
resolution of stenosis.
There is no clinical trial data to support antiplatelet therapy over antico-
agulation, and clinical practice varies. Warfarin is the typical treatment for
secondary stroke prevention from dissection and is usually given for three
to six months, but ASA may be used as an alternative. Since recanalization
can occur in the first few months, we typically perform noninvasive repeat
imaging to reassess the stenosis.
This patient was started on ASA and eventually treated with three
months of warfarin. Three months later, she had only residual right facial
palsy but was otherwise neurologically normal. Imaging showed the left
internal carotid artery (LICA) remained persistently occluded, but she had
robust collaterals (Figures 17.3 and 17.4). Her warfarin was stopped, and
she was started on ASA. She has had no recurrent events.

FIGURE 17.2 Angiogram showing tapering occlusion of the left ICA. Mechanical thrombectomy
of the distal ICA allowed for cross-filling from the right.

17 CAROTID DISSECTION 75
FIGURE 17.3 MRI with patchy left MCA infarct.

FIGURE 17.4 MRA showing persistent occlusion of the left ICA but robust collateral flow to the
LMCA from the anterior communicating artery.

76 WHAT DO I DO NOW? CEREBROVASCULAR DISEASE


KEY POINTS

■ Stroke and a partial Horner’s syndrome should prompt an


investigation for dissection.
■ Antithrombotic therapy is typically used for stroke prevention with
either warfarin or aspirin.
■ Recurrent stroke rates are low after dissection on medical therapy.

Bibliography
Kremer C, Mosso M, Georgiadis D, et al. Carotid dissection with permanent and transient
occlusion or severe stenosis: Long term outcome. Neurology. 2003;60:271–275.
Patel R, Adam R, Maldjian C, et al. Cervical carotid artery dissection: Current review of
diagnosis and treatment. Cardiol in Rev. 2012;20:145–152.
Schievink WI. Spontaneous dissection of the carotid and vertebral arteries. N Engl J Med.
2001;344:898–906.

17 CAROTID DISSECTION 77
This page intentionally left blank
18 Stroke in a Young Adult

A 36-year-old woman with a history of migraine


presented with headache and blurred vision. She initially
thought her visual symptoms were related to migraine,
but her symptoms worsened and she came to the ER.
She was found to be neurologically normal in the ER. An
MRI was done, and it showed small, scattered temporal
parietal infarcts (Figures 18.1 and 18.2).
Her initial labs showed a thrombocytopenia and
anemia. Her transesophageal echocardiogram (TEE)
was normal.

What do you do now?

79
FIGURES 18.1–18.2 MRI DWI sequence showing recent bilateral infarcts in the temporal and
parietal lobes.

S troke in young adults is rare. The definition of young in various studies


has been usually under 45 or 55. This is an age where traditional
vascular risk factors can develop, and these should definitely be investigated.
The prevalence of hypertension, diabetes, and high cholesterol has been
increasing in young adults.
There are other risk factors that seem to be more prominent in young
people. Dissection is a common cause of stroke in young. Cardiac sources of
emboli are also detected in young adults, and these include prosthetic heart
valves, endocarditis, cardiomyopathy, and cardiac tumors. Patent foramen
ovale and atrial septal defects are also found more commonly in young
patients with stroke. Other potential risk factors identified in young adults
are smoking, migraine with aura, pregnancy and puerpurium, oral contra-
ceptive use, and illicit drug use. An evaluation for stroke etiology should
include screening for these and is easily done by history. Vessel imaging and
thorough cardiac evaluation are important.
There are numerous very rare causes of stroke and these should be con-
sidered in the proper clinical context. Patients should be asked about sys-
temic symptoms and other organ system involvement. A family history of
venous or arterial thromboses is also important to obtain some indication
of a hereditary thrombophilia. Evaluation for these more exotic causes of
stroke require more specialized testing.

80 WHAT DO I DO NOW? CEREBROVASCULAR DISEASE


TABLE 18.1 Uncommon causes of stroke
Infectious
Endocarditis: erythrocyte sedimentation rate (ESR), complete blood count
(CBC), TEE, blood cultures
Tuberculosis: CSF tuberculosis polymerase chain reaction (PCR), Purified
Protein Derivative (PPD), chest x-ray (CXR)
Syphilis: rapid plasma reagin (RPR), CSF
Varicella zoster virus (VZV): CSF PCR
Human Immunodeficiency Virus (HIV): serological testing
Bacterial meningitis: CSF culture
Autoimmune
Primary central nervsous system (CNS) angiitis: CSF cells and protein, brain
biopsy
Systemic autoimmune diseases:
Systemic lupus erythematosus
Sjogren’s
Behcet’s
Sarcoidosis
Inflammatory bowel disease
Genetic:
Factor V Leiden
Prothrombin gene mutation
Methylene tetrahydrofolate reductase (MTHFR) mutation
Cerebral autosomal dominant arteriopathy with subcortical infarcts and
leukoencephalopathy (CADASIL): NOTCH 3
Fabry’s disease: alpha galactosidase activity
Hematological:
Protein C, S
Antithrombin III
Hyperhomocysteinemia
Sickle cell disease: hemoglobin electropheresis
Disseminated intravascular coagulation (DIC)
Thrombotic thrombocytopenic purpura (TTP)

This patient was young and had no significant risk factors for stroke.
Her initial workup included evaluation for more typical causes of stroke.
Her TEE was normal. She had an angiogram that showed no evidence of
large-vessel vasculopathy. The key laboratory findings were the low platelets
and anemia. A peripheral smear confirmed hemolytic anemia, and she was
diagnosed with thrombotic thrombocytopenic purpura (TTP), a rare cause
of stroke. She was treated with plasma exchange.

18 STROKE IN A YOUNG ADULT 81


KEY POINTS

■ Stroke in young adults is rare.


■ Other, more unusual causes of stroke need to be investigated in
the absence of traditional risk factors.
■ There are multiple autoimmune, infectious, and genetic disorders
that can (rarely) cause stroke.

Bibliography
Ferro JM, Massaro AR, Mas JL. Aetiological diagnosis of ischaemic stroke in young
adults. Lancet Neurol. 2010;9:1085–1096.
Larrue V, Berhoune N, Massabuau P. et al. Etiologic investigation of ischemic stroke in
young adults. Neurology. 2011;76:1983–1988.

82 WHAT DO I DO NOW? CEREBROVASCULAR DISEASE


19 Mycotic Aneurysm Due to
Bacterial Endocarditis

A 74-year-old woman with hypertension, atrial


fibrillation, and mechanical aortic and mitral valves
presented with headache and confusion. A CT scan
showed intracerebral hemorrhage and a small amount
of subarachnoid hemorrhage (Figures 19.1, 19.2, 19.3
and 19.4).
Her INR is 2.3 and she is now neurologically normal.
She was noted to have a fever with a temperature of
101.9 degrees.

What do you do now?

83
FIGURES 19.1–19.4 CT with small parasagittal intraparenchymal hemorrhage and small
subarachnoid hemorrhage.

A patient with fever, mechanical valves, and a brain hemorrhage


should raise strong suspicion of bacterial endocarditis. Neurological
complications of bacterial endocarditis can be seen in 25% of patients.
These complications include ischemic stroke, subarachnoid hemorrhage
(SAH), ICH, meningitis, abscess formation, and encephalopathy.
Stroke occurs from embolization of septic material from the infected
heart valve. Intraparenchymal hemorrhage is more commonly due to hemor-
rhagic transformation of an ischemic infarct. However, rupture of a mycotic
aneurysm may also cause ICH or SAH. Mycotic aneurysms are relatively
rare and found in 2%–10% of patients with infective endocarditis. Mycotic
aneurysms arise from infective embolic debris that causes local inflammation
of the vessel. Unlike berry aneurysms, mycotic aneurysms are not typically

84 WHAT DO I DO NOW? CEREBROVASCULAR DISEASE


found at the circle of Willis since the emboli migrate distally in the arterial
tree. For this reason, they are more difficult to detect on MRA or CTA.
Acute neurological deficit and abnormalities on imaging should be fol-
lowed up with vessel imaging. Noninvasive imaging such as MRA or CTA
may be used initially. But because of their small size and distal location,
mycotic aneurysms are not excluded by normal findings on MRA or CTA,
and conventional angiography is needed. In this patient, since she would
need to be anticoagulated again for the mechanical valves, it is especially
important to evaluate for an aneurysm. We also evaluate for aneurysms in
patients who need valve replacement since they are aggressively anticoagu-
lated during surgery.
Best treatment for mycotic aneurysms is not clear. There are no random-
ized clinical trials. Based on several case series and expert opinion, ruptured
aneurysms are usually treated with endovascular embolization or surgery.
Unruptured aneurysms are treated medically with antibiotics and with care-
ful serial monitoring for growth. If the lesion remains stable, antibiotics are
continued, but if there is growth, treatment with embolization or surgery
is considered.
This patient had a TEE that showed no vegetations. Blood cultures grew
S. viridans. She was started on antibiotics, and her warfarin was held. A CTA
was done, which showed a distal ACA aneurysm (Figures 19.5 and 19.6).

FIGURE 19.5 CTA coronal view showing small left ACA aneurysm.

19 MYCOTIC ANEURYSM DUE TO BACTERIAL ENDOCARDITIS 85


FIGURE 19.6 CTA sagittal view with distal left anterior cerebral artery (LACA) aneurysm.

Because this patient had a hemorrhage and she would eventually need
to resume anticoagulation for her mechanical heart valves, she was taken
for treatment with embolization (Figures 19.7 and 19.8). Her warfarin
was held for one week, then resumed after embolization. She continued IV

FIGURE 19.7 Cerebral angiogram lateral view of distal aneurysm.

86 WHAT DO I DO NOW? CEREBROVASCULAR DISEASE


FIGURE 19.8 Angiogram post-embolization of aneurysm.

antibiotics for six weeks. A repeat angiogram at the end of her antibiotic
course showed no new lesions.

KEY POINTS

■ Fever and stroke or hemorrhage in a patient with mechanical


valves should prompt an evaluation for bacterial endocarditis.
■ Mycotic aneurysms are rare but can cause hemorrhage.

Bibliography
Heiro M, Nikoskelainen J, Engblom E, et al. Neurological manifestations of infective
endocarditis: a 17-year experience in a teaching hospital in Finland. Arch Intern Med.
2000;160:2781–2787.
Peters PJ, Harrison T, Lennox JL. A dangerous dilemma: management of
infectious intracranial aneurysms complicating endocarditis. Lancet Infect Dis.
2006;6:742–748.

19 MYCOTIC ANEURYSM DUE TO BACTERIAL ENDOCARDITIS 87


This page intentionally left blank
20 Migrainous Stroke

A 48-year-old man with a history of migraine since


age 13 presented with visual symptoms and left-sided
weakness. His migraines typically start with peripheral
vision loss and seeing “jellyfish” throughout the visual
field. This is usually followed by pulsatile headache.
They occur four times a year. The morning of admission,
he noticed his typical visual aura. However, this was
followed by left-sided weakness in addition to his usual
headache. He went to the ER. His visual symptoms
resolved within 30 minutes. The weakness improved
significantly, but he was found to have a mild arm drift.
He continued to have headache.
An MRI done acutely showed a region of restricted
diffusion in the right corona radiata (Figure 20.1).

What do you do now?

89
FIGURE 20.1 MRI DWI with evidence of infarct along the right lateral ventricle.

T his patient is having a migraine but has a new focal neurological deficit.
As discussed in a prior chapter, migraine may be a stroke mimic, but
migraine may also be a risk factor for stroke. There are many proposed
mechanisms linking migraine to stroke: recurrent vasodilation may lead to
impaired cerebral autoregulation; endothelial change can occur; increased
inflammation can cause increased platelet reactivity. Migraine has been
associated with stroke in young people.
Acute migraine can also directly cause stroke. The International Headache
Society (IHS) defines migrainous stroke as a typical attack of migraine with
aura, not reversible in seven days, and with evidence of infarct on imaging.
A study of patients with stroke in the setting of migraine included patients
with migraine with or without aura, presenting with acute migraine and
neurological symptoms, and who were found to have acute infarct on MRI
DWI. The majority (70%) had DWI lesions in the posterior circulation.
Almost all (94%) had at least one other risk factor for stroke, especially
hypertension. The exact mechanism of migraine directly causing stroke is
unknown, but it is speculated that sustained oligemia or vasospasm with
thrombosis during aura plays a role.
Migrainous stroke would be a diagnosis of exclusion. This patient had a
full evaluation of his vessels and heart. He had no evidence of a large artery
or cardiac source of his stroke. He had no hematological abnormalities.

90 WHAT DO I DO NOW? CEREBROVASCULAR DISEASE


Since this patient had evidence of evolving infarct on imaging with symp-
toms occurring in the setting of a migraine, we diagnosed him with migrain-
ous stroke. Although there are no clinical trial data to support this, migraine
prophylaxis was initiated to reduce the frequency of migraine, as a stroke
preventative strategy. He was also started on ASA, and his medical risk fac-
tors were monitored.

KEY POINTS

■ Migraine may be a risk factor for stroke.


■ Migrainous stroke is a rare occurrence, and thorough evaluation of
other etiologies is needed.

Bibliography
Etminan M, Takkouche B, Isorna FC, Samii A. Risk of ischaemic stroke in people with
migraine: systematic review and meta-analysis of observational studies. BMJ.
2005;330:63.
Kruit MC, van Buchem MA, Hofman PA, et al. Migraine as a risk factor for subclinical brain
lesions. JAMA. 2004;291:427–434.
Stang PE, Carson AP, Rose KM, et al. Headache, cerebrovascular symptoms, and stroke:
The Atherosclerosis Risk in Communities Study. Neurology. 2005;64:1573–1577.
Swartz RH, Kern RZ. Migraine is associated with magnetic resonance imaging white
matter abnormalities. Arch Neurol. 2004;61:1366–1368.

20 MIGRAINOUS STROKE 91
This page intentionally left blank
21 Dural Venous Sinus
Thrombosis

A 40-year-old man with no significant medical history


developed altered mental status while driving a bus. He
had been well until a week prior to admission, when he
developed flu-like symptoms. He had nausea, vomiting,
and a band-like headache. On the day of admission, he
was driving a bus and felt unusual. He missed his turn
and hit a pole. He does not remember anything after
this. He was brought into the ER and was witnessed to
be combative. He had a generalized tonic clonic seizure
in the ER. A CT scan showed a right frontal ICH and a
small right sylvian SAH. There was hyperdensity in the
region of the right transverse sinus and superior sagittal
sinus (Figures 21.1, 21.2, 21.3 and 21.4).

What do you do now?

93
FIGURE 21.1 CT showing hyperdensity along the right transverse sinus.

T he findings on the noncontrast head CT are suggestive of dural venous


sinus thrombosis. The hyperdensities in the area of the transverse
sinus and superior sagittal sinus strongly indicate this. He also has cortical
hemorrhages that may be due to venous congestion.
Dural venous sinus thrombosis is a rare disease in which thrombosis of
intracranial veins and/or sinuses occurs. Patients are younger (mean age, in
one series, of 39) and more commonly female. The most common causes

FIGURE 21.2 Right sylvian subarachnoid blood and hyperdensity in the superior sagittal sinus.

94 WHAT DO I DO NOW? CEREBROVASCULAR DISEASE


FIGURES 21.3–21.4 Small hemorrhages in the right frontal lobe.

are prothrombotic state, such as hereditary thrombophilia, pregnancy or


puerperium, and oral contraceptive use. Infections such as otitis or mas-
toiditis can cause local inflammation and thrombosis as well. This is more
common in children. Head trauma and lumbar puncture have also been
suggested as risk factors for venous thrombosis.
Patients most commonly present with headache, which may be acute or
subacute in onset. Patients may develop altered mental status, and seizures
occur in 40% of patients. Imaging may show venous infarcts with hemor-
rhage. Initial CT findings may show hyperdensity in the region of the sinus

21 DURAL VENOUS SINUS THROMBOSIS 95


(as in this patient), and contrast studies may show an “empty delta” sign
in which there is no contrast opacification of the confluence of sinuses.
Diagnosis is usually confirmed by magnetic resonance venogram (MRV) or
computed tomography venogram (CTV).
Treatment is usually with anticoagulation. Clinicians are often nervous
about this since there may be intracerebral hemorrhage on CT. But, it must
be kept in mind that the hemorrhage is due to venous thrombosis and
back pressure from poor venous drainage. The treatment of the underly-
ing thrombosis should reduce the risk of recurrent hemorrhage. There are
no large randomized trial data to support this. However, in a large regis-
try with over 600 patients, 80% were treated with anticoagulation show-
ing usual clinical practice. There was 8% mortality in this study, and risk
factors for poor outcome included being male, age over 37, coma, ICH,
deep-venous-system involvement, and cancer.
This patient had an MRI and MRV that confirmed dural venous sinus
thrombosis (DVST). He was placed on low-molecular-weight heparin and
transitioned to warfarin. He was also placed on leviteracetam for seizures.
He recovered fully. He had a full hypercoagulable evaluation, which revealed
a homozygous mutation of MTHFR C677. He continued warfarin for a
year, then was switched to ASA. He remained symptom-free and without
recurrent thrombosis.

KEY POINTS

■ Headache and hemorrhagic infarcts in patients who are


hypercoagulable (including pregnancy and puerperium) should
prompt an evaluation for dural venous sinus thrombosis.
■ Treatment is typically with anticoagulation, despite hemorrhage
on imaging.

Bibliography
Ferro JM, Canhao P, Stam J, Bousser MG, Barinagarrementeria F. Prognosis of cerebral
vein and dural sinus thrombosis: results of the International Study on Cerebral Vein
and Dural Sinus Thrombosis (ISCVT). Stroke. 2004;35:664–670.
Stam J. Thrombosis of the cerebral veins and sinuses. N Engl J Med. 2005;352:1791–1798.

96 WHAT DO I DO NOW? CEREBROVASCULAR DISEASE


22 Moyamoya Disease

A 43-year-old woman developed recurrent neurological


events. She developed intermittent episodes of left face
and arm tingling and clumsiness of the left hand. She
also had dysarthria. These episodes were recurrent and
lasted an hour. She was seen at another hospital and
was found to have multiple infarcts in both hemispheres.
She had an angiogram and was told she had vasculitis.
She was treated with corticosteroids, which precipitated
depression and mania, requiring a psychiatric
hospitalization. She was placed on multiple antipsychotic
medications and subsequently transferred to us. She
was tangential and grandiose but was neurologically
normal. Her imaging showed the following features seen
in Figures 22.1, 22.2 and 22.3.

What do you do now?

97
FIGURE 22.1 MRI with bilateral chronic infarcts in a distal ICA territory.

T his patient was having recurrent TIAs. Her pattern of infarcts on MRI is
suggestive of distal ICA watershed infarcts. Her MRA showed bilateral
distal ICA occlusions. This was confirmed by angiography. In a young patient
with bilateral ICA occlusive disease, Moyamoya disease should be considered.
This type of occlusive disease is not consistent with CNS vasculitis.
Moyamoya is a disease in which there is bilateral stenosis or occlusion
of the terminal portion of the ICAs. An abnormal vascular network of col-
lateral blood vessels develops at the base of the brain that has an angio-
graphic appearance of a puff of smoke. There is a higher prevalence in Asian
populations, but it can be seen in other race/ethnic groups. Moyamoya
has been associated with cranial radiation, Down’s syndrome, and sickle
cell disease. Pathologically, the vessels appear to develop intimal thickening
with smooth-muscle proliferation.
Patients may present with stroke or hemorrhage, with hemorrhage more
common in adults and rarer in children. Headaches and seizures may also
occur. Medically, patients are usually treated with antiplatelet therapy to
reduce the risk of stroke from thrombi at areas of stenoses. Anticoagulation
is not used, especially since there is a risk of ICH from rupture of friable
collateral vessels.
There are no clinical trials to assess different treatments, but different revas-
cularization techniques have been used, mostly in children. There appears

98 WHAT DO I DO NOW? CEREBROVASCULAR DISEASE


FIGURE 22.2 MRA with occlusion of both ICAs.

to be benefit with surgical treatment, but there are limited data in adults.
Since the disease is limited to intracranial vessels, the goal is to redirect exter-
nal carotid artery branches to supply the ischemic brain. Direct bypass can
be done with a superficial temporal artery to MCA bypass allowing diver-
sion of extracranial blood to the intracranial circulation. This allows for rapid

FIGURE 22.3 Cerebral angiogram showing occlusion of the ICA and prominent lenticulostriate
vessels.

22 MOYAMOYA DISEASE 99
restoration of blood flow. Indirect bypass with encephaloduroarteriosynan-
giosis (EDAS) allows for ECA branches to develop collateral flow to the isch-
emic brain over time.
In this patient, hemodynamic studies confirmed flow failure. She ini-
tially had a right-sided EDAS followed two months later by left EDAS. She
had no recurrent strokes.

KEY POINTS

■ Moyamoya is a rare cause of stroke in the United States.


■ Angiographic findings of distal ICA stenosis with lenticulostriate
collaterals are important in diagnosis.
■ Treatment, especially in young patients, is with surgical
revascularization.

Bibliography
Pandey P, Steinberg GK. Neurosurgical advances in the treatment of Moyamoya disease.
Stroke. 2011;42:3304–3310.
Scott RM, Smith ER. Moyamoya disease and Moyamoya syndrome. N Engl J Med.
2009;360:1226–1237.

100 WHAT DO I DO NOW? CEREBROVASCULAR DISEASE


23 Reversible Cerebral
Vasoconstriction Syndrome

A 46-year-old woman with only high cholesterol


presented with the worst headache of her life while
traveling in Asia. She developed acute holocephalic
headache that radiated down into the neck. The
headache was maximal in intensity at onset. She took
two aspirins with minimal relief. She went to a local clinic
and was found to have an SBP of 170. A head CT was
reportedly normal, and she was given pain medications.
She returned to the United States three days later and
saw a neurologist, who sent her to the ER. She was
neurologically normal. Her imaging showed a small
amount of SAH on CT, which was confirmed on MRI
(Figures 23.1 and 23.2).

What do you do now?

101
FIGURES 23.1–23.2 MRI FLAIR showing hyperintensities in sulci consistent with subarachnoid
hemorrhage.

T his presentation of thunderclap headache needs to be evaluated


emergently for aneurysmal SAH. Her imaging, however, is not
suggestive of aneurysmal hemorrhage, since the distribution of blood
is patchy and not in a typical location for a berry aneurysm rupture. An
angiogram is warranted to evaluate for vascular anomalies. Her angiogram
showed areas of focal stenoses (Figure 23.3).
This pattern of focal stenoses is highly suggestive of reversible cerebrova-
soconstriction syndrome. Historically, this disorder was called Call-Fleming

102 WHAT DO I DO NOW? CEREBROVASCULAR DISEASE


FIGURE 23.3 Cerebral angiogram showing focal stenoses primarily in the ACA branches.

syndrome, and early reports were in women during pregnancy or puerpe-


rium. Migraines, medications such as selective serotonin reuptake inhibi-
tors (SSRIs), and sympathomimetic drugs have also now been associated
with the disorder. The diagnosis is based on clinical history and imaging
findings. There are no serological or CSF markers of the disease. The diag-
nosis is confirmed by reversal of the angiographic abnormality.
In a case series of patients with reversible cerebral vasoconstriction syn-
drome (RCVS), the authors found a younger group with mean age of 42,
and there were more women. Eighty-one percent had brain lesions: isch-
emic stroke, convexity SAH, lobar ICH, and brain edema. Calcium chan-
nel blockers and glucocorticoids have been used in this disorder, but it is
unclear if there is an effect on outcomes. The prognosis is usually good.
This patient had a very benign course with resolution of headaches. She
was placed on verapamil, a calcium channel blocker. She had a follow-up
angiogram four weeks later that showed reversal of vasoconstriction.

23 REVERSIBLE CEREBRAL VASOCONSTRICTION SYNDROME 103


KEY POINTS

■ Severe “thunderclap” headaches warrant an immediate evaluation


for subarachnoid hemorrhage.
■ Reversible vasoconstriction syndrome is associated with
pregnancy and puerperium but may occur spontaneously or in
association with medications or migraine.
■ Empiric steroids and calcium channel blockers have been used, but
there are no data to support this.

Bibliography
Singhal AB, Hajj-Ali RA, Topcuoglu MA, et al. Reversible cerebral vasoconstriction
syndromes: Analysis of 139 cases. Arch Neurol. 2011;68:1005–1012.

104 WHAT DO I DO NOW? CEREBROVASCULAR DISEASE


24 Transient Ischemic Attack
Diagnosis and Management

A 56-year-old man with a history of metastatic prostate


cancer and tobacco abuse presented with two episodes
of right-sided weakness. He developed acute numbness
and weakness on the right and was unable to talk.
He fell because of the weakness. Symptoms lasted 10
minutes and resolved completely. One hour later, he had
recurrence of the same symptoms, which again resolved.
He came to the ER and was found to have a normal
examination. His blood pressure was 120/88. His CT was
normal. He is unable to get an MRI because of metal in
his body. The patient would like to go home.

What do you do now?

105
T his patient had a TIA. He had acute neurological symptoms that fit
a vascular territory. The definition of TIA has been evolving. The
traditional definition was time-based, and symptoms had to resolve within
24 hours. The 24-hour time limit was arbitrary and certainly did not fit
with timing of irreversible brain infarction. A newer definition includes a
shorter duration of symptoms and includes imaging. This definition of TIA
is: symptoms typically last less than an hour, and there is no evidence of
infarct on imaging. There have been attempts at risk-stratification of TIA
patients. The ABCD2 score is commonly used (Table 24.1).
This scale may help identify patients who are at high risk of early recur-
rence. In one study, the risk of recurrent stroke or TIA at 48 hours with
the highest ABCD2 scores was up to 8%. Imaging may also be of use.
Studies have suggested that patients who have clinical resolution of symp-
toms but show evidence of acute infarct on MRI may also be at high risk
of recurrence.
Regardless of score, we generally admit all patients with TIA to expe-
dite a workup. The workup should include brain imaging, vascular imag-
ing, and cardiac evaluation, with at least an EKG and an echocardiogram.
Hospitalization is usually the quickest way to complete an evaluation
and start a prevention program. However, other means of rapid evalu-
ation and treatment, such as a TIA clinic, have been shown to improve
outcomes.
This patient was hospitalized for an evaluation that included an echocar-
diogram and vascular imaging. He was found to have a LMCA stenosis and
was treated with antiplatelets and statin.

TABLE 24.1 ABCD2 score for patients with TIA


ABCD2 score
A. Age over 60: 1
B. Blood pressure 140/90: 1
C. Clinical features weakness: 2, speech disturbance without weakness: 1,
other: 0
D. Duration >60 min: 2, 10–59 min: 1, <10 min: 0
E. Diabetes: 1

106 WHAT DO I DO NOW? CEREBROVASCULAR DISEASE


KEY POINTS

■ Patients with TIA are at risk of having a recurrent TIA or stroke.


■ Patients need to be evaluated quickly and secondary stroke
prevention strategies put in place, since the highest risk period is
soon after a TIA.

Bibliography
Arsava EM, Furie KL, Schwamm LH, et al. Prediction of early stroke risk in transient
symptoms with infarction: relevance to the new tissue based definition. Stroke.
2011;42:2186–2190.
Galvin R, Geraghty C, Motterlini N, et al. Prognostic value of the ABCD2 clinical prediction
rule: a systematic review and meta-analysis. Fam Pract. 2011;28:366–376.
Johnston SC, Rothwell PM, Nguyen-Huynh MN, et al. Validation and refinement of
scores to predict very early stroke risk after transient ischaemic attack. Lancet.
2007;369:283–292.
Rothwell PM, Giles MF, Chandratheva A, et al. Effect of urgent treatment of transient
ischaemic attack and minor stroke on early recurrent stroke (EXPRESS study): a
prospective population-based sequential comparison. Lancet. 2007;370:1432–1442.
Rothwell PM, Giles MF, Flossmann E, et al. A simple score (ABCD) to identify individuals at
high early risk of stroke after a transient ischaemic attack. Lancet. 2005;366:29–36.

24 TIA DIAGNOSIS AND MANAGEMENT 107


This page intentionally left blank
25 Hypertensive Intracerebral
Hemorrhage

A 65-year-old woman with a history of hypertension and


atrial fibrillation developed acute left leg numbness. She
was on warfarin for atrial fibrillation but admitted to
poor compliance. She noticed numbness in the left foot
when she was walking barefoot and did not feel the cold
tiles like she felt them with the right foot. She denied
any headache, visual change, or motor symptoms. She
saw her doctor, who requested an outpatient head CT.
The CT revealed a hemorrhage, and she was sent to the
ER. In the ER, her blood pressure was 205/78. She had
mild weakness in the left arm and leg and dense sensory
loss in the left leg. Her INR was normal. Her CT showed a
hemorrhage in the right putamen (Figure 25.1).

What do you do now?

109
FIGURE 25.1 Head CT with hyperdensity in the right basal ganglia suggestive of hypertensive ICH.

T he location and appearance of the hematoma is suggestive of


hypertensive ICH. She has a history of hypertension and had a
significantly elevated blood pressure on presentation. The typical locations
for hypertensive hemorrhage include basal ganglia, pons, cerebellum, and
thalamus. The immediate treatment should be to lower the blood pressure.
There is not much evidence to support specific blood pressure targets and
rate of blood pressure drop in patients with ICH. AHA guidelines for
hypertensive hemorrhage recommend lowering blood pressure to a target
of 160/90 if there are no intracranial pressure (ICP) issues. There are some
data to support the conclusion that reducing the SBP to 140 is safe. In
these situations, it is important to use a rapid-acting and easily titratable
medication for blood pressure control. Intravenous drips such as nicardipine
are often used.
She had been on warfarin but was noncompliant. In patients with coag-
ulopathy and hemorrhage, coagulopathy should be corrected rapidly. In
patients on warfarin, this may be done with IV vitamin K, fresh frozen
plasma, or prothrombin complex concentrate. The benefit of platelet trans-
fusions in patients on antiplatelet agents is unclear, but they are often admin-
istered, especially in patients with persistent bleeding. Anticonvulsants are
not recommended for prophylaxis unless there is high risk of seizures, and
ICP is an issue since seizure can increase ICP. Deep venous thrombosis

110 WHAT DO I DO NOW? CEREBROVASCULAR DISEASE


(DVT) prophylaxis is important, and after 48 hours, if there is no hema-
toma expansion, subcutaneous heparin may be started.
Prevention of recurrent hemorrhage, especially in a patient with hyper-
tensive hemorrhage, is to aggressively control blood pressure to normoten-
sion. Resumption of anticoagulation in patients with hemorrhage requires
weighing the risks and benefits. Anticoagulation in lobar ICH is usually not
recommended, since the likelihood of recurrent hemorrhage is higher with
amyloid angiopathy. With small, deep hemorrhages such as this, the risk
of recurrent hemorrhage needs to be balanced against the risk of ischemic
stroke from atrial fibrillation.
In this patient, anticoagulation was held for three months. Follow-up
imaging showed resolution of the hematoma, and her blood pressure was
well controlled. Her gradient echo (GRE) sequence showed no evidence
of other microhemorrhages on MRI that would indicate possible amyloid
angiopathy. Her warfarin was resumed for atrial fibrillation.

KEY POINTS

■ Patients with suspected ICH should be immediately evaluated with


CT scanning.
■ If coagulopathic, correction of coagulopathy must be instituted
rapidly.
■ Blood pressure should be monitored and treated, with a goal SBP
of under 160 when ICP is not elevated.

Bibliography
Adeoye O, Broderick JP. Advances in the management of intracerebral hemorrhage. Nat
Rev Neurol. 2010;6:593–601.
Gurol ME, Greenberg SM. Management of intracerebral hemorrhage. Curr Atheroscler
Rep. 2008;10:324–331.
Morgenstern LB, Hemphill JC, Anderson C, et al. Guidelines for the management of
spontaneous intracerebral hemorrhage. A guideline for healthcare professionals
from the American Heart Association/American Stroke association. Stroke. 2010;41.
2108–2129.

25 HYPERTENSIVE ICH 111


This page intentionally left blank
26 Cerebral Amyloid Angiopathy

A 69-year-old woman with prior intracerebral


hemorrhages and dementia presented with decreased
verbal output and somnolence. She had initially been
seen seven years prior to presentation with a right
temporal ICH. The hemorrhage was surgically evacuated,
and pathology revealed cerebral amyloid angiopathy.
She had a right frontal lobar hemorrhage a year after
the first hemorrhage. She remained stable and was
ambulatory and conversant but required 24-hour
assistance because of cognitive difficulty for the next
few years. She now returns with garbled speech. Her
blood pressure was 177/72. She was found to be awake
and mumbling incoherently. She did not follow any
commands. She moved all limbs spontaneously. A CT
scan showed a new lobar hemorrhage in the left sylvian
region (Figure 26.1).

What do you do now?

113
FIGURE 26.1 CT showing skull defect due to prior craniotomy, encephalomalacia in the right
frontal and temporal lobes, hyperdensity in the left temporal/insular lobes.

C erebral amyloid angiopathy (CAA) needs to be considered in a patient


with multiple lobar hemorrhages. CAA is an important cause of ICH
in the elderly. It is caused by beta amyloid deposition in brain vessels. This
deposition can lead to small-vessel occlusion and white matter disease. The
vessels are also friable and can rupture, causing hemorrhage. Diagnosis can
only be made pathologically.
The Boston Criteria have been developed to categorize probable and pos-
sible CAA. Definite CAA is based on pathological confirmation. Probable
CAA with pathology includes patients who have had a lobar ICH and CAA
is found in the pathological specimen. Probable CAA is in patients with
multiple microhemorrhages in lobar or cortico-subcortical locations and of
an age over 55. Possible CAA is in patients with a single lobar ICH and an
age of over 55. This patient had pathological confirmation of CAA upon
first presentation. Her subsequent hemorrhages have also all been lobar.
She had surgical evacuation of her first hemorrhage several years ago.
Although this is not standard therapy, the International Surgical Trial in
Intracerebral Hemorrhage (STICH) trial showed non-significant trend
toward benefit in patients who had lobar hemorrhage near the surface.
Evacuation of clot may be considered in such patients. But given this patient’s
diagnosis and her severely disabled state, surgery was not considered.

114 WHAT DO I DO NOW? CEREBROVASCULAR DISEASE


Unfortunately, there is no treatment for CAA. Patients are medically
managed to lower the risk of recurrent hemorrhage. Anticoagulants are
avoided in patients with CAA. The risks and benefits of antiplatelet therapy
need to be weighed carefully. Although there are no trials for blood pres-
sure control in patients with CAA, patients with ICH in general have lower
recurrence rates with aggressive blood pressure treatment, so this should be
instituted in patients with CAA as well.

KEY POINTS

■ Lobar ICH in an elderly patient may be due to cerebral amyloid


angiopathy.
■ Imaging findings of cortical microhemorrhages on MRI may
support this diagnosis.
■ There is no specific medical therapy to prevent hemorrhages in
CAA.

Bibliography
Mendelow AD, Gregson BA, Fernandes HM, et al.; STICH investigators. Early surgery
versus initial conservative treatment in patients with spontaneous supratentorial
intracerebral haematomas in the International Surgical Trial in Intracerebral
Haemorrhage (STICH): a randomised trial. Lancet. 2005;365(9457):387–397.
Smith EE, Greenberg SM. Clinical diagnosis of cerebral amyloid angiopathy: validation of
the Boston Criteria. Curr Atheroscler Rep. 2003;5:260–266.
Viswanathan A, Greenberg SM. Cerebral amyloid angiopathy in the elderly. Ann Neurol.
2011;70:871–880.

26 CEREBRAL AMYLOID ANGIOPATHY 115


This page intentionally left blank
27 Intracerebral Hemorrhage
Secondary to Arteriovenous
Malformation

A 49-year-old man with no past medical history


presented to the ER with vertigo, gait unsteadiness,
nausea, and vomiting, followed by progressive lethargy.
His mental status rapidly declined in the ER, and he
was emergently intubated. Initial CT scanning showed
hemorrhage in the cerebellum and intraventricular
hemorrhage (Figure 27.1). A CTA showed abnormal
dilated vessels.

What do you do now?

117
FIGURE 27.1 CT with hemorrhage in the cerebellar vermis and fourth ventricle.

A young person without a history of hypertension presenting with a


spontaneous ICH should be evaluated for an underlying lesion. The
abnormal vessels on CTA are indicative of arteriovenous malformation
(AVM) rupture as an etiology of this hemorrhage. AVMs are a tangle of
vessels with abnormal shunting directly from the arterial to the venous
system without intervening capillaries. AVMs are relatively rare, with a
detection rate about 1.2 to 1.4 per 100,000 persons per year. Some AVMs
may remain asymptomatic. They can also present with seizure. About half
of patients with AVM present with hemorrhage. Some epidemiological
studies suggest a risk of first hemorrhage of 2%–4% per year. Risk factors
for hemorrhage include small size, deep drainage, high intranidal pressure,
and presence of a feeding artery aneurysm. After a hemorrhage, the risk of
rebleeding was found in one study to be 18% per year. Although treatment
for asymptomatic AVMs is controversial, given the high rebleeding rate after
first hemorrhage, treatment of a ruptured AVM is typically warranted.
The Spetzler-Martin Scale is commonly used to assess surgical risk for
treatment (Table 27.1).
Higher grade lesions were associated with greater morbidity with sur-
gery. This scale was designed to address outcomes with surgery, but it has
also been applied to other types of treatment.
Treatment includes surgical resection, embolization in which embolic mate-
rial is delivered into feeding arteries, radiosurgery, or a combination of these.
There have been no trials to show benefit of any modality over another.

118 WHAT DO I DO NOW? CEREBROVASCULAR DISEASE


TABLE 27.1 Spetzler-Martin Scale
Size:
0–3 cm 1
3.1–6.0 2
>6 cm 3
Location:
Non-eloquent 0
Eloquent 1
Deep venous drainage:
Not present 0
Present 1

In this patient, an external ventricular drain (EVD) was placed for


the intraventricular blood. His mental status improved. An angiogram
was then done, which showed a vermian AVM with a dysplastic aneu-
rysm of PICA (Figure 27.2). This vessel was embolized (Figure 27.3).
The patient then had a suboccipital decompression and AVM resection.
Follow-up angiography showed no residual AVM. He continued to have
residual ataxia.

FIGURE 27.2 Cerebral angiogram showing PCA and PICA feeding arteries to the cerebellar AVM.

27 INTRACEREBRAL HEMORRHAGE SECONDARY TO AVM 119


FIGURE 27.3 Cerebral angiogram after embolization and resection.

KEY POINTS

■ A young patient without hypertension should have an evaluation


for the underlying cause of ICH.
■ AVM is a (rare) cause of ICH.
■ Treatment after hemorrhage may include embolization, surgery,
and radiosurgery.

Bibliography
Ogilvy CS, Stieg PE, Awad I, et al. Special Writing Group of the Stroke Council,
American Stroke Association. AHA Scientific Statement: recommendations for
the management of intracranial arteriovenous malformations: a statement for
healthcare professionals from a special writing group of the Stroke Council. Stroke.
2001;32:1458–1471.
Stapf C, Mast H, Sciacca RR, et al. Predictors of hemorrhage in patients with untreated
brain arteriovenous malformation. Neurology. 2006;66:1350–1355.
van Beijnum J, van der Worp HB, Buis DR. et al. Treatment of brain arteriovenous
malformations: a systematic review and meta-analysis. JAMA. 2011;306:2011–2019.

120 WHAT DO I DO NOW? CEREBROVASCULAR DISEASE


28 Intracerebral Hemorrhage
(ICH) from Cavernous
Malformation

A 20-year-old college student developed new-onset


headache. He had no medical problems. A month prior
to presentation, he noticed sharp occipital headaches.
They were severe and lasted a few minutes. They
occurred five times a day. Over the course of a week,
the headaches evolved into a dull, constant pain that
was relieved with ibuprofen. He eventually had a head
CT that showed a hemorrhage. He was neurologically
normal and had a normal blood pressure. He then had an
MRI (Figure 28.1).

What do you do now?

121
FIGURE 28.1 MRI T2 weighted sequence showing chronic hemosiderin in the right temporal lobe.

T he lesion in the right temporal lobe of mixed ages of blood is highly


suggestive of a cavernous malformation.
Cavernous malformations are circumscribed clusters of capillaries. With
the advent of widespread MRI imaging, they are being detected more com-
monly. They are estimated to occur in about one in 200 people. About 20%
of these are familial, with an autosomal-dominant mutation in one of three
identified genes: CCM1, CCM2, and CCM3. The rest are sporadic. These
lesions can cause symptoms of headache, seizure, and, depending on loca-
tion, focal neurological symptoms. Hemorrhage can be devastating.
Diagnosis is usually by MRI. As in this patient, different ages of blood in
a circumscribed lesion can be seen. These lesions are angiographically occult,
but an angiogram may be done to exclude an arteriovenous malformation.
When should such lesions be treated? This is a controversial area, and
there are no randomized trials to guide management. To approach this
rationally, one must understand the natural history and risk of hemorrhage
before considering treatment.
Unfortunately, the studies evaluating risk of hemorrhage from cavernous
malformations are mixed because of heterogeneity in the study methodolo-
gies. The risks appear to range between 0.25 to 4.5% per year for recurrent
hemorrhage. Lesion location seems to be important, with deep lesions more
likely to bleed. Cavernous malformations associated with developmental
venous anomalies also may have a higher bleeding risk.

122 WHAT DO I DO NOW? CEREBROVASCULAR DISEASE


A recent study followed 292 patients with known cavernous malforma-
tion over time to determine risk factors and rates of hemorrhage. In this
study, patients who initially presented with a hemorrhage had a 6% annual
rate of hemorrhage compared with 0.33% in patients whose cavernous mal-
formation was detected incidentally. The median time to second hemor-
rhage was eight months, and the risk declined after the first two years. In
this series, 80% of patients who presented with hemorrhage had no recur-
rent hemorrhage for 10 years.
Surgical intervention is considered when there is progressive neurologi-
cal deficit due to recurrent hemorrhage, or for intractable epilepsy. In deep
and eloquent regions, the risk of hemorrhage needs to be weighed against
the morbidity associated with surgery, which may be as high as 10%.
Stereotactic radiosurgery has also been used in patients with surgically inac-
cessible lesions. The efficacy of this intervention in reducing hemorrhage
rates is not well defined.
This patient had an angiogram that revealed no underlying AVM. He had
evidence of a developmental venous anomaly, which is often seen in associa-
tion with cavernous malformations. Because he had no neurological symptoms
despite evidence of hemorrhage, he was managed conservatively. He under-
stood the risk of possible seizure and hemorrhage. We monitored for clinical
symptoms, and he remained symptom-free out to eight years of follow-up.

KEY POINTS

■ Cavernous malformations may cause symptoms due to recurrent


bleeding.
■ Intervention should be considered in patients with progressive
neurological symptoms or refractory epilepsy.
■ Cavernous malformations may be familial.

Bibliography
Batra S, Rigamonti K, Rigamonti D. Management of hemorrhage from cavernous
malformations. Curr Atheroscler Rep. 2012;14:360–365.
Del Curling Jr O, Kelly Jr DL, Elster AD, et al. An analysis of the natural history of
cavernous angiomas. J Neurosurg. 1991;75:702–708.
Flemming KD, Link MJ, Christianson TJH, Brown RD. Prospective hemorrhage risk of
intracerebral cavernous malformations. Neurology. 2012;78:632–636.

28 ICH FROM CAVERNOUS MALFORMATION 123


This page intentionally left blank
29 Aneurysmal Subarachnoid
Hemorrhage

A 43-year-old woman with a history of tobacco abuse


and hypertension controlled with medications presented
to the ER with sudden-onset severe, throbbing,
global, persistent headache followed by syncope. On
evaluation in the ER, her vitals were normal and she
was neurologically intact without signs of meningismus
or focal neurological deficits. Initial CT scan was
interpreted as normal (Figures 29.1, 29.2, 29.3 and 29.4).

What do you do now?

125
FIGURES 29.1–29.4 CT without obvious subarachnoid blood, but hyperdensity along the
tentorium.

A cute worst headache of one’s life requires immediate evaluation for


aneurysmal subarachnoid hemorrhage. A diagnosis of SAH cannot
be missed. The mortality is high (50%), and the risk of rebleeding after the
initial hemorrhage is also high (15%–20% within the first two weeks). CT
scan done acutely has about 95% sensitivity for detecting subarachnoid
blood. The location of blood can indicate the location of the ruptured
aneurysm. For example, MCA aneurysms may have a focal clot within
the Sylvian fissure. ACA aneurysms may have more blood within the
interhemispheric fissure. Often, however, blood is distributed throughout
the supracellar cistern and cannot provide a clue to aneurysm location.

126 WHAT DO I DO NOW? CEREBROVASCULAR DISEASE


In this patient, the subarachnoid component of the hemorrhage was not
obvious on CT. The hemorrhage in this case is more subdural and can be
seen along the tentorium. Clinical suspicion for SAH with a negative head
CT, however, requires a lumbar puncture to evaluate for xanthochromia,
which would confirm hemorrhage. In this patient, an MRI was immediately
done, which showed subdural and subarachnoid hemorrhage as well as the
location of the aneurysm (P comm.) (Figures 29.5, 29.6, 29.7 and 29.8).
The Hunt Hess grading system is a commonly used tool to quantify
the deficit. This score also has prognostic implications, with a higher score
associated with worse outcome.

FIGURES 29.5–29.8 FLAIR with subtle subarachnoid and subdural blood. 29.8: MRI with
contrast, coronal view showing left posterior communicating artery aneurysm.

29 ANEURYSMAL SUBARACHNOID HEMORRHAGE 127


HUNT HESS:

1. Asymptomatic, mild headache, mild meningismus


2. Moderate–severe headache, nuchal rigidity, no neurological deficit
except cranial nerve palsy
3. Drowsiness or confusion, mild neurological deficit
4. Stupor, moderate to severe hemiparesis
5. Coma, decerebrate posturing
Our patient had a Hunt Hess grade 1, suggesting a good prognosis.
Once the diagnosis is made, patients should be monitored closely in a
neurological ICU. This allows for frequent neurological exams, hemody-
namic monitoring, and management by a team familiar with subarachnoid
hemorrhage treatment. Because of the risk of rebleeding, the aneurysm
should be treated as quickly as possible. While awaiting treatment, the
patient’s blood pressure should be monitored closely and treated to keep
the SBP<160 to lower the risk of rebleeding. If the aneurysm cannot be
treated quickly, short-term use of aminocaproic acid can be initiated to help
prevent rebleeding.
Treatment is either surgical clipping of the aneurysm or endovascu-
lar coiling. Choice of treatment strategy depends on the anatomy of the
aneurysm (location, size, aneurysm neck size) and the patient’s risk fac-
tors. In general, endovascular treatment seems to have lower morbidity and
mortality.
After treatment, there is risk of vasospasm. Subarachnoid blood can cause
local vessel irritation, causing spasm. This can lead to stroke. Vasospasm
usually occurs seven to 10 days after a hemorrhage. Patients are typically
treated prophylactically with a calcium channel blocker, nimodipine, and
hydration to maintain euvolemia. Screening transcranial Dopplers are done
to evaluate for increasing velocities that may suggest developing spasm. If
symptomatic vasospasm develops, hypertensive therapy or angioplasty may
be considered.
In this patient, an angiogram confirmed the presence of a bilobed pos-
terior communicating artery aneurysm with a narrow neck projecting
posterolaterally. Coil embolization of the aneurysm was done without com-
plications (Figures 29.9 and 29.10). The patient had an uneventful course
in the neurological ICU and did not develop any symptomatic vasospasm.

128 WHAT DO I DO NOW? CEREBROVASCULAR DISEASE


FIGURES 29.9 Cerebral angiogram showing bilobed Pcomm aneurysm.

FIGURE 29.10 Cerebral angiogram after coiling of aneurysm.

29 ANEURYSMAL SUBARACHNOID HEMORRHAGE 129


She made a complete recovery and was discharged home with no neurologi-
cal symptoms. Follow-up angiograms done at six months and at one year
showed no evidence of recurrence or recanalization of the aneurysm.

KEY POINTS

■ Sudden severe headache requires rapid evaluation for SAH.


■ A negative CT with high clinical suspicion for SAH warrants a
lumbar puncture.
■ Morbidity and mortality are high from aneurysmal SAH, so
treatment needs to be initiated quickly.

Bibliography
Connolly ES, Rabinstein AA, Carhuapoma JR, et al. Guidelines for the management of
aneurysmal subarachnoid hemorrhage. A guideline for healthcare professionals
from the American Heart Association/American Stroke association. Stroke.
2012;43:1711–1737.

130 WHAT DO I DO NOW? CEREBROVASCULAR DISEASE


30 Perimesencephalic
Subarachnoid Hemorrhage

A 58-year-old woman with no medical problems


developed a sudden “worst headache of her life” while
working as nurse. A CT scan showed SAH (Figure 30.1).
She was neurologically normal but had severe headache.
She had an angiogram that showed no aneurysm.

What do you do now?

131
FIGURE 30.1 CT scan with subarachnoid hemorrhage in the suprasellar cistern.

A s discussed in the prior cases, worst headache of life requires evaluation


for SAH. In this patient, SAH was present. An angiogram was
performed and did not reveal an aneurysm.
In about 15% of patients with SAH, no aneurysm is detected. About
two-thirds of these non-aneurysmal SAH patients have a pattern of per-
imesencephalic SAH. This is usually defined as hemorrhage in the basal
cisterns, usually anterior to the pons and midbrain and with little exten-
sion into the sylvian fissures or interhemispheric fissure. This localization
is important because the prognosis is quite different between perimesen-
cephalic and non-perimesencephalic SAH. The etiology of hemorrhage is
suspected to be venous, but this has not been definitively shown. In large
case series, patients with perimesencephalic SAH had little to no risk of
rebleeding or vasospasm. On the other hand, in non-perimesencephalic
non-aneurysmal SAH, there was a significant risk of rebleed, vasospasm,
and hydrocephalus.
An MRI was done to evaluate for possible thrombosed aneurysm since
an angiogram will not opacify a thrombosed aneurysm. We also evaluated
for a spinal AVM with an MRI. Bleeding from a cervical dural AVM may
cause hemorrhage upward into the basal cisterns. These were normal. The
diagnosis of perimesencephalic SAH is made after a thorough evaluation
for aneurysm or AVM has been completed. This patient had an extensive

132 WHAT DO I DO NOW? CEREBROVASCULAR DISEASE


workup for possible source of hemorrhage. We typically do a conven-
tional angiogram and repeat it in 10–14 days to evaluate for a recanalized
aneurysm.

KEY POINTS

■ About 15% of patients with SAH have no obvious cause of


bleeding.
■ These patients should have a thorough evaluation for aneurysm or
other source of hemorrhage.
■ Perimesencephalic SAH generally has a good prognosis.

Bibliography
Canovas D, Gil A, Jato M, et al. Clinical outcome of spontaneous non-aneurysmal
subarachnoid hemorrhage in 108 patients. Eur J Neurol. 2012;19:457–461.
Rinkel GJ, Wijdicks EF, Hasan D, et al. Outcome in patients with subarachnoid
hemorrhage and negative angiography according to pattern of hemorrhage on
computed tomography. Lancet. 1991;338:961–968.

30 PERIMESENCEPHALIC SAH 133


This page intentionally left blank
31 Asymptomatic Intracranial
Aneurysm

A 41-year-old man with a history of high cholesterol and


active tobacco abuse was well until two months ago.
He noted a gradual onset of right-sided headache. He
described the headache as “pressure.” He had no nausea
or vomiting but did report bilateral blurred vision and
photophobia. The headache is daily and constant but he
has relief with ibuprofen. He has had no motor, sensory,
or language symptoms. His neurological examination is
normal. He had an MRI and MRA of the brain to evaluate
this new-onset headache. It revealed a 5 mm aneurysm
of the right internal carotid artery siphon.

What do you do now?

135
W ith the widespread use of brain imaging such as MRI and MRA
for symptoms like headache, we are detecting more asymptomatic
intracranial aneurysms. The knowledge that there is 50% mortality with
rupture of an aneurysm provokes much anxiety. Patients naturally want to
know their risk of bleeding and the potential treatments.
In order to decide about any treatment for an intracranial aneurysm that
has not bled, it is important to evaluate the risk of bleeding and weigh that
against the risks and benefit of treatment. The location and size of the aneu-
rysm is important, since both play a significant role in risk of hemorrhage.
The first question in location is whether the aneurysm is intradural or extra-
dural. Cavernous carotid aneurysms are extradural and hence do not cause
subarachnoid hemorrhage when they rupture. They can cause symptoms
due to compression of other structures. Eye movement abnormalities can
be seen due to compression of the third, fourth, and sixth cranial nerves.
Treatment of these aneurysms is usually warranted only if they cause com-
pressive symptoms.
Carotid aneurysms that are intradural cause subarachnoid hemorrhage.
The next level of determining risk is whether the aneurysm is within the
anterior or posterior circulation. Then the size needs to be considered.
The International Study of Unruptured Intracranial Aneurysms (ISUA)
study found that, in patients without a history of prior ruptured aneurysm,
the cumulative risk of SAH in five years was dependent on location and size
(see Table 31.1).
A recent Japanese study also showed a relationship between size and loca-
tion with risk of hemorrhage. They found A comm and P comm aneurysms
more likely to rupture. This study also found morphology of the aneurysm
was important. Aneurysms with a daughter sac were also more likely to
rupture.

TABLE 31.1 5 year hemorrhage rates of intracranial aneurysms by size and


location

Location <7 mm 7–12 13–24 >25


Anterior 0 2.6% 14.5% 40%
Posterior 2.5% 14.5% 18.4% 50%

From Wiebers D. Unruptured intracranial aneurysms: natural history, clinical outcome, and risks
of surgical and endovascular treatment. Lancet. 2003;362(9378):103–110.

136 WHAT DO I DO NOW? CEREBROVASCULAR DISEASE


Once risk of hemorrhage is estimated, we then need to weigh the risks
and benefits of treatment. The two options for treatment are surgical aneu-
rysm clipping and endovascular coiling. There are no randomized trials com-
paring these two techniques for unruptured aneurysms. The International
Subarachnoid Aneurysm Trial (ISAT) was a trial comparing clipping and
coiling in patients who had hemorrhage. They found one-year morbidity and
mortality higher in the surgically treated patients. In one non-randomized
study of outcomes in patients treated with clipping and coiling, poor out-
come (death or discharge to nursing home or rehabilitation facility) was seen
in 13% of those treated surgically versus 7% of those treated endovascularly.
In-hospital mortality was also higher with surgery. Patient age over 50 was
associated with worse outcomes in surgically treated patients. Overall,
it appears endovascular treatment is associated with better outcomes, but
future randomized trials in unruptured aneurysms are needed.
For this patient, the distinction of intra- versus extradural location of the
aneurysm was not clear on noninvasive imaging. Sizing is also more accu-
rate on a conventional angiogram. For these reasons, this patient had an
angiogram to determine the exact location. He was found to have a 3.4 mm
intradural hypophyseal aneurysm.
The small size in a favorable location is associated with low risk of hem-
orrhage. He was counseled to stop smoking since this is a risk factor for
hemorrhage. He was also advised to monitor his blood pressure and ensure
that it would be treated if needed. He had a follow-up MRA at a year and
no enlargement was detected. He remained symptom-free.

KEY POINTS

■ Symptomatic large aneurysms should be considered for


treatment.
■ Asymptomatic aneurysms in a patient who has had prior SAH should
be considered for treatment since risk of hemorrhage is higher.
■ Small aneurysms may be monitored.
■ Treatment needs to be individualized. Surgery has been
associated with neurological disability and cognitive impairment.
Endovascular coiling may be associated with a higher rate of
incomplete obliteration. There are no randomized trial data.

31 ASYMPTOMATIC INTRACRANIAL ANEURYSM 137


Bibliography
Bederson JB, Awad IA, Wiebers DO, et al. Recommendations for the management of
patients with unruptured intracranial aneurysms. A statement for healthcare
professionals from the Stroke Council of the American Heart Association. Stroke.
2000;31:2742–2750.
Higashida RT, Lahue BJ, Torbey MT, et al. Treatment of unruptured intracranial
aneurysms: a nationwide assessment of effectiveness. Am J Neuroradiol.
2007;28:146–151.
International Study of Unruptured Intracranial Aneurysms Investigators. Unruptured
intracranial aneurysms: natural history, clinical outcome, and risks of surgical and
endovascular treatment. Lancet. 2003;362:103–110.
The UCAS Japan Investigators. The natural course of unruptured cerebral aneurysms in
a Japanese cohort. N Engl J Med. 2012;366:2474–2482.

138 WHAT DO I DO NOW? CEREBROVASCULAR DISEASE


Index

Note: Page numbers in italics indicate illustrations.

abscess formation, and bacterial mycotic vs. berry aneurysms, 84–85


endocarditis, 84 post-embolization, 87
ACAS aneurysmal subarachnoid hemorrhage,
See Asymptomatic Carotid 125–130
Atherosclerosis Study (ACAS) hospitalization and treatment, 128
acute ischemic stroke Hunt Hess grading system for
conservative medical therapy for, 10 neurological deficit, 127, 128
endovascular treatment for, 7–11 images, 126, 127
key points, 11 morbidity and mortality rates, 126, 130
options available, 8 angioplasty
IV tPA treatment for, 1–5 after stroke due to carotid stenosis, 58
benefits of, 2–3, 3–4 in ICA stenosis, 50–51, 51
inclusion and exclusion criteria, 2–4, 3 angiotensin converting enzyme (ACE)
key points, 5 inhibitors
patients with minor symptoms, 28–29 primary prevention of stroke, 45
and managing hypertension, 39–41 secondary stroke prevention, 54
primary prevention of, 43–47 angiotensin receptor blockers (ARBs)
rapid improvement as ominous sign, 28 primary stroke prevention, 45
reperfusion therapy for, 13–20 secondary stroke prevention,
risk-assessment scales, 46 53, 55
risk factors for, 45 anhydrosis, and Horner’s syndrome, 74
signs of, 1, 7, 8, 13 anticoagulation therapy
African-Americans avoiding in patients with cerebral
response to ACE inhibitors, 45 amyloid angiopathy (CAA), 115
risk of intracranial atherosclerosis, 62 for dural venous sinus thrombosis, 96
aminocaproic acid, 128 list of oral anticoagulants, 67
aneurysm resuming in patients with hemorrhage,
asymptomatic intracranial aneurysm, 111
135–138 secondary stroke prevention, 75
complications and likelihood of antiplatelet therapy
rupture, 136 after stenting, 59
treatment risk and benefits, 137 intracranial atherosclerosis and stroke
bilobed, 129 prevention, 62–63
distal aneurysm, 86 in patients with cerebral amyloid
evaluation and treatment for, 85 angiopathy (CAA), 115
left ACA aneurysm, 85, 86 primary stroke prevention, 46

139
antiplatelet therapy (cont.) bacterial endocarditis, and mycotic
secondary stroke prevention, 53, 54, 55, 75 aneurysm, 83–87
for transient ischemic attack (TIA), 106 basal gangllia calcifications, 2, 22
to treat Moyamoya disease, 98 beta amyloid deposition, effects of,
apixaban, 68 114
arteriovenous malformation (AVM) Boston Criteria to categorize cerebral
images, 119, 120 amyloid angiopathy, 114
incidence, 118 brainstem, and compression from edema,
and intracerebral hemorrhage (ICH), 32, 37, 38
117–120 “bridging therapy” and recanalization, 16
treatment, 118–119
ASA calcifications, basal ganglia, 2, 22
See aspirin prophylaxis for stroke calcium channel blockers
Asian-Americans after aneurysmal subarachnoid
prevalence of Moyamoya disease, 98 hemorrhage, 128
risk of intracranial atherosclerosis, 62 to treat reversible cerebral
aspirin prophylaxis for stroke vasoconstriction syndrome, 103
and intracranial atherosclerosis, 62–63 Call-Fleming syndrome, 101–104
prevention of secondary stroke cardioembolic stroke
after cryptogenic stroke, 71 incidence in young adults, 80
after dissection, 75, 77 secondary stroke prevention after,
after lacunar stroke, 53, 54, 55 65–68
after migrainous stroke, 91 carotid aneurysms, 136
after thrombosis, 96 carotid dissection, 73–77
primary stroke prevention, 46 carotid endarterectomy
Asymptomatic Carotid Atherosclerosis after stroke due to carotid stenosis, 58
Study (ACAS), 50 and asymptomatic stenosis, 50
asymptomatic intracranial aneurysm, 135–138 perioperative complications, 58–59
complications and likelihood of rupture, Carotid Revascularization Endarterectomy
136 Versus Stenting Trial (CREST), 50, 58
treatment risk and benefit, 137 carotid stenosis, and secondary stroke
asymptomatic stenosis prevention, 57–59
of internal carotid artery (ICA), 49–52 cavernous carotid aneurysms,
angiography, 51 136
angioplasty, 50–51 cavernous malformation
carotid endarterectomy, 50 image suggestive of, 122
medical management of, 50, 51, 52 and intracerebral hemorrhage, 121–123
atorvastatin, 54, 55 cerebellar stroke
atrial fibrillation, and risk for stroke, 66, MRI diffusion-weight sequence showing,
68, 70–71 36, 37
atrial septal aneurysm, and cryptogenic rapid deterioration after, 37
stroke, 70 suboccipital decompression for, 35–38

140 INDEX
cerebral amyloid angiopathy (CAA), 113–115 Down’s syndrome, and Moyamoya
cerebral autoregulation and perfusion disease, 98
pressure, 40 drain, ventriculostomy, 38
cerebral vasoconstriction syndrome, dural venous sinus thrombosis, 93–96
reversible, 101–104 diagnosis, 95–96
cervical carotid artery, dissection in previous images suggestive of, 94
trauma patient, 74 risk factors for, 94–95
CHADS2 score, and stroke risk from atrial treatment, 96
fibrillation, 66, 68
cholesterol, in young adults, 80 ECASS
clopidogrel See European Cooperative Acute Stroke
genetic variants in hepatic metabolism, Study (ECASS)
54–55 edema
secondary stroke prevention, 54, 55, 63 causes and treatment, 32–33
CLOSURE trial, 70 and compression of right lateral
coronary disease, and treating hypertension, 45 ventricle, 32
cortico-subcortical hemorrhages, 114, 115 Ehler-Danlos type IV, 74
cranial radiation, and Moyamoya disease, encephalopathy, and bacterial
98 endocarditis, 84
craniectomy endarterectomy, carotid
hemicraniectomy, 31–34 after stroke due to carotid stenosis, 58
suboccipital craniectomy, 38 and asymptomatic stenosis, 50
CREST perioperative complications, 58–59
See Carotid Revascularization endocarditis, bacterial, 83–87
Endarterectomy Versus Stenting endovascular treatment
Trial (CREST) for acute ischemic stroke, 7–11
cryptogenic stroke, and secondary stroke key points, 11
prevention after, 69–71 options available, 8
epilepsy, and cavernous
dabigitran, 68 malformation, 123
deep vein thrombosis, prevention after etiology of stroke in young adults, 81
ICH, 110–111 European Cooperative Acute Stroke Study
developmental venous anomaly, 123 (ECASS)
diabetes defining symptomatic hemorrhagic
increasing prevalence in young adults, 80 transformation, 23
and secondary stroke prevention, 63, 71 treating acute ischemic stroke with IV
and treating hypertension, 45, 46 tPA, 3, 4, 5
dipyridamole, extended release, 54 exercise, and stroke prevention, 63
dissection extended-release dipyridamole, 54
as cause of stroke in young adults, 80 external ventricular drain (EVD),
of internal carotid artery, 73–77 119
diuretics, 45, 54 extradural vs. intradural aneurysm, 136

INDEX 141
fever, and bacterial endocarditis, 83, 84 predictors of after tPA, 23
fibromuscular dysplasia, and spontaneous secondary to arteriovenous
dissection, 74 malformation (AVM), 117–120
focal stenoses, 103 secondary to cavernous malformation,
Framingham Stroke Profile, 46 121–123
fresh frozen plasma, 110 symptomatic vs. asymptomatic, 23
frontal cortex, punctate infarcts in, perimesencephalic subarachnoid
28, 29 hemorrhage, 131–133
subarachnoid hemorrhage in
glucocorticoids, 103 suprasellar cistern, 132
vs. non-perimesencephalic, 132
head trauma hemorrhagic complications of tPA, 4, 8, 9,
and risk of dural venous sinus 21–24
thrombosis, 95 hemorrhagic transformation
and use of IV tPA, 75 gradient echo sequence, 66
headache and infarcted tissue, 22
migraine, 25–26 post-tPA, 22
“thunderclap headache,” 101, 102, 104, symptomatic vs. asymptomatic, 23
132 usual practice in treating post-tPA, 23
heart valves, and mycotic aneurysm, heparin
83–87 low-molecular-weight, 96
hemicraniectomy subcutaneous, 111
for large MCA stroke, 31–34 hepatic metabolism, genetic variants in,
quality of life after, 33, 34 54–55
scan showing, 33 hormone replacement therapy, 46
hemorrhage Horner’s syndrome, 73, 74, 77
aneurysmal subarachnoid hemorrhage, Hunt Hess grading system for neurological
125–130 deficit, 127, 128
hospitalization and treatment, 128 hydrocephalus, and cerebellar stroke, 37, 38
Hunt Hess grading system for hyperosmolar therapy, and reducing brain
neurological deficit, 127, 128 edema, 32, 33
images, 126, 127 hypertension
morbidity and mortality rates, 126, 130 association with lacunar stroke, 54
in cerebellar vermis and fourth ventricle, association with migrainous stroke, 90
118 increasing prevalence in young adults, 80
hypertensive intracerebral hemorrhage, management in acute stroke, 39–41
109–111 management in stroke prevention
images of, 84 and cryptogenic stroke, 71
intracerebral hemorrhage (ICH) and intracranial atherosclerosis, 63
hypertensive ICH, 109–111 as modifiable risk factor for stroke, 45
and intravenous tissue plasminogen monitoring and treating, 46
activator (IV tPA), 4, 21–24 post IV tPA monitoring of, 4

142 INDEX
treating hypertensive ICH, 110, 111 secondary to arteriovenous malformation
treating patients with cerebral amyloid (AVM), 117–120
angiopathy, 115 incidence, 118
hypertensive intracerebral hemorrhage, treatment, 118–119, 120
109–111 secondary to cavernous malformation,
hypertonic saline, and reducing brain 121–123
edema, 32, 33, 38 symptomatic vs. asymptomatic, 23
hypothermia, and reducing brain edema, 33 intracranial aneurysm, asymptomatic,
135–138
infarct, images of complications and likelihood of rupture,
along right lateral ventricle, 90 136
bilateral chronic infarcts in a distal ICA treatment risk and benefit, 137
territory, 98 intracranial atherosclerosis, and secondary
left frontal, 66 stroke prevention, 61–63
patchy LMCA, 62 intradural vs. extradural aneurysm, 136
recent bilateral infarcts, 80 intraparenchymal hemorrhage, 84
right temporal and insular, 58 intravenous tissue plasminogen activator
internal carotid artery (ICA) (IV tPA)
asymptomatic stenosis, 49–52 hemorrhagic complications of, 4, 8, 9,
Moyamoya disease, 97–100 21–24
occlusion of both ICA, 99 hemorrhagic transformation after, 22
occlusion of left ICA, 74 hypertension monitoring and treatment,
persistent occlusion with robust collateral 4, 13–14
flow, 76 risk for intracerebral hemorrhage (ICH), 4
tapering occlusion of left ICA, 75 treating acute ischemic stroke, 1–5
international normalized ratio (INR), in benefits of, 2–3, 3–4
secondary stroke prevention, 65, 68 inclusion and exclusion criteria, 2–4,
International Study of Unruptured 3, 8, 13
Intracranial Aneurysms (ISUA), treating patients with minor symptoms,
136 28–29
International Subarachnoid Aneurysm Trial treating stroke mimic, 26
(ISAT), 137 use in patients with dissection, 75
International Surgical Trial in Intracerebral using lower dose of in “bridging therapy,” 16
Hemorrhage (STICH), 114 ISAT
intra-arterial prourokinase (proUK), 8 See International Subarachnoid
intra-arterial thrombolysis, and MCA Aneurysm Trial (ISAT)
occlusion, 8 ISUA
intracerebral hemorrhage (ICH) See International Study of Unruptured
hypertensive ICH, 109–111 Intracranial Aneurysms (ISUA)
and intravenous tissue plasminogen IV tPA
activator (IV tPA), 4, 21–24 See intravenous tissue plasminogen
predictors of after tPA, 23 activator (IV tPA)

INDEX 143
Labetolol, 13, 40 Mechanical Embolus Removal in Cerebral
lacunar stroke, 53–56 Ischemia (MERCI) Study, 8, 16
left-middle cerebral artery meningitis, and bacterial endocarditis,
(LMCA) 84
dense left MCA sign, 74 middle cerebral artery (MCA)
density in, 14, 15 hemicraniectomy for large MCA stroke,
infarct, 76 31–34
infarct and hemorrhagic transformation, infarct and hemorrhagic transformation,
22 22
infarct and stenosis, 62 infarction of complete RMCA territory,
signs of occlusion, 7, 8 32
stem occlusion, 16 left-middle cerebral artery (LMCA)
stenosis and transient ischemic attack, density in, 14, 15
106 signs of occlusion, 7, 8, 14
lenticulostriate vessels, 99 stem occlusion, 16
leviteracetam, 96 limited infarcts, 19
lisinopril, 40, 45 occlusion, 9
lobar hemorrhages, 114, 115 migraine headache
lumbar puncture and reversible cerebral vasoconstriction
diagnosing subarachnoid hemorrhage, syndrome, 103
127, 130 and signs of stroke, 25–26
and risk of dural venous sinus migrainous stroke, 89–91
thrombosis, 95 miosis, and Horner’s syndrome, 74
Moyamoya disease, 97–100
Management of Atherothrombosis with mycotic aneurysm due to bacterial
Clopidogrel in High Risk Patients endocarditis, 83–87
(MATCH) trial, 55
mannitol National Institute of Neurological Disorders
for cerebellar stroke, 35 and Stroke (NINDS)
and reducing brain edema, 32, 33 defining symptomatic hemorrhagic
Marfan’s syndrome, and spontaneous transformation, 23
dissection, 74 treating patients with minor symptoms,
mastoiditism, and risk of dural venous sinus 28
thrombosis, 95 trial of IV tPA, 2–3
MATCH trial nicardipine, 110
See Management of Atherothrombosis nimodipine, 128
with Clopidogrel in High Risk North American Symptomatic Carotid
Patients (MATCH) trial Endarterectomy Trial (NASCET),
mechanical clot retrieval 58
following IV tPA, 16
and MCA occlusion, 8–9 oral contraceptive use, and risk of dural
recanalization following, 17 venous sinus thrombosis, 95

144 INDEX
otitis, and risk of dural venous sinus proton pump inhibitors (PPIs), and
thrombosis, 95 interaction with clopidogrel, 54
proUK
paradoxical embolus, and cryptogenic See intra-arterial prourokinase (proUK)
stroke, 70 ptosis, and Horner’s syndrome, 74
parasagittal intraparenchymal hemorrhage, 84 puerperium
patent foramen ovale (PFO) and reversible cerebral vasoconstriction
and cryptogenic stroke, 69, 70, 71 syndrome, 103
and stroke in young adults, 80 and risk of dural venous sinus
Patient Foramen Ovale in Cryptogenic thrombosis, 95
Stroke Study (PICSS), 70 punctate infarcts in left frontal cortex, 28, 29
Penumbra device for clot retrieval, 8–9
perfusion pressure, 40 radiosurgery
perimesencephalic subarachnoid for cavernous malformation, 123
hemorrhage, 131–133 treating arteriovenous malformation
prognosis vs. non-perimesencephalic, (AVM), 118, 120
132 recanalization
subarachnoid hemorrhage in suprasellar after mechanical thrombectomy, 17
cistern, 132 “bridging therapy,” 16
PICSS and carotid artery dissection, 75
See Patient Foramen Ovale in and intravenous tissue plasminogen
Cryptogenic Stroke Study (PICSS) activator (IV tPA), 15, 15–16
pregnancy and mechanical clot retrieval, 8–9
and reversible cerebral vasoconstriction of occluded MCA branch artery, 10
syndrome, 103 occlusion and, 18
and risk of dural venous sinus reperfusion therapy for acute ischemic
thrombosis, 95 stroke, 13–20
preventing stroke, 43–47 revascularization
evaluation for risk factors, 45 and carotid stenosis, 50–51,
silent infarcts, risk factors and incidence, 58, 59
44 to treat Moyamoya disease, 98–100
Prevention Regimen for Effectively reversible cerebral vasoconstriction
Avoiding Second Strokes syndrome, 101–104
(PROFESS) trial, 54 right lateral ventricle, compression of, 32
primary prevention of stroke, 43–47 right middle cerebral artery (RMCA)
evaluation for risk factors, 45 hemicraniectomy for large MCA stroke,
silent infarcts, risk factors and incidence, 31–34
44 infarction of complete RMCA territory,
Prolyse in Acute Cerebral 32
Thromboembolism Trial risk-assessment scales for stroke
(PROACT), 8 ABCD2 for transient ischemic attack
prothrombin complex concentrate, 110 (TIA), 106, 106

INDEX 145
risk-assessment scales for stroke (cont.) SPS3 trial
CHADS2, 66, 68 See Secondary Prevention of Small
Framingham Stroke Profile, 46 Subcortical Strokes (SPS3) trial
Spetzler-Martin Scale and arteriovenous statins
malformation, 118, 119 for intracranial atherosclerosis, 63
risk factors, evaluation for, 44–45, 80, 82 secondary stroke prevention
rivaroxaban, 68 after cryptogenic stroke, 71
ruptured vs. unruptured aneurysm, 136 after lacunar stroke, 54, 55
for transient ischemic attack (TIA), 106
SAMMPRIS trial stenosis
See Stenting and Aggressive Medical asymptomatic of internal carotid artery
Management for Preventing (ICA), 49–52
Recurrent Stroke in Intracranial angiography of, 51
Stenosis (SAMMPRIS) trial angioplasty, 50–51
Secondary Prevention of Small Subcortical carotid endarterectomy, 50
Strokes (SPS3) trial, 55 medical management of, 50, 51, 52
secondary stroke prevention and carotid artery dissection, 75
after cardioembolic stroke, 65–68 carotid stenosis and secondary stroke
after cryptogenic stroke, 69–71 prevention, 57–59
after lacunar stroke, 53–56 focal stenoses, 103
after stroke due to carotid stenosis, left MCA, 62
57–59 and risk of recurrent stroke, 62
after stroke due to intracranial Stenting and Aggressive Medical
atherosclerosis, 61–63 Management for Preventing
after subarachnoid hemorrhage, 128 Recurrent Stroke in Intracranial
after transient ischemic attack (TIA), Stenosis (SAMMPRIS) trial, 63
106, 107 stenting and angioplasty
selective serotonin reuptake inhibitors angiography, 51
(SSRIs), 103 in ICA stenosis, 50–51
sickle cell disease, and Moyamoya disease, 98 perioperative stroke, 59
silent infarcts, risk factors and incidence, 44 secondary stroke prevention
simvastatin, 53 after stroke due to carotid stenosis, 58
smoking cessation, and stroke prevention, 63 after stroke due to intracranial
sociodemographic factors in treating atherosclerosis, 63
hypertension, 45 STICH trial
Solitaire With the Intention For See International Surgical Trial in
Thrombectomy Trial (SWIFT), 8–9 Intracerebral Hemorrhage (STICH)
SPARCL trial stroke
See Stroke Prevention by Aggressive incidence in United States, vii
Reduction of Cholesterol Levels managing hypertension, 39–41
(SPARCL) trial migrainous stroke, 89–91
Spetzler-Martin Scale, 118, 119 primary prevention, 43–47

146 INDEX
rapid improvement as ominous sign, 28 suboccipital craniectomy, 38
risk-assessment scales, 46 suboccipital decompression
risk factors for, 45 for cerebellar stroke, 35–38
secondary stroke prevention treating intracerebral hemorrhage,
after cardioembolic stroke, 65–68 119
after cryptogenic stroke, 69–71 SWIFT
after lacunar stroke, 53–56 See Solitaire With the Intention for
after stroke due to carotid stenosis, Thrombectomy Trial (SWIFT)
57–59 Sylvian fissure, hyperdensity in, 14, 15
after stroke due to intracranial sympathomimetic drugs, 103
atherosclerosis, 61–63
after subarachnoid hemorrhage, 128 thrombectomy
silent infarcts, risk factors and incidence, following IV tPA, 16
44 and MCA occlusion, 8–9
stroke in a young adult, 79–82 recanalization following, 17
stroke mimic thrombophilia, hereditary, 95
migraine headache, 25–26, 90 thrombosis, dural venous sinus,
treatment with IV tPA, 26 93–96
treating minor symptoms, 27–29 thrombotic thrombocytopenic purpura
Stroke Prevention by Aggressive Reduction (TTP), 81
of Cholesterol Levels (SPARCL) “thunderclap headache,” 101, 102, 132
trial, 54 transient ischemic attack (TIA),
subarachnoid hemorrhage (SAH) 105–107
hyperintensities in sulci consistent with
SAH, 102 vasoconstriction syndrome, reversible
small SAH, 84 cerebral, 101–104
subarachnoid hemorrhage, aneurysmal, vasospasm, 128
125–130 venous anomaly, developmental,
hospitalization and treatment, 128 123
Hunt Hess grading system for ventriculostomy drain, 38
neurological deficit, 127, 128 verapamil, 103
images, 126, 127 vitamin K, intravenous, 110
morbidity and mortality rates, 126, vomiting, and dissection of cervical carotid
130 artery, 74
subarachnoid hemorrhage,
perimesencephalic, 131–133 warfarin
prognosis vs. non-perimesencephalic, and patients with hypertensive ICH,
132 110, 111
subarachnoid hemorrhage in secondary stroke prevention
suprasellar cistern, 132 after cardioembolic stroke, 65, 68
and “thunderclap” headaches, 102, 104 after cryptogenic stroke, 70
subcutaneous heparin, 111 after stroke from dissection, 75, 77

INDEX 147
warfarin (cont.) weight loss, and stroke prevention, 63
in intracranial atherosclerosis, 62–63 white matter hyperintensities, 44
treating dural venous sinus thrombosis,
96 young adults
withholding therapy, 85, 86 migraine headache and risk of stroke,
Warfarin-Aspirin Symptomatic Intracranial 90
Disease Trial (WASID), 62 as stroke patients, 79–82

148 INDEX

Vous aimerez peut-être aussi